This commit is contained in:
Josia Pietsch 2023-07-28 03:45:37 +02:00
parent 25e248c605
commit 296c7b2f55
Signed by: josia
GPG Key ID: E70B571D66986A2D
23 changed files with 267 additions and 198 deletions

View File

@ -66,7 +66,7 @@ Define $\cF = \bigcup_{n \in \N} \cF_n$. Then $\cF$ is an algebra.
We'll show that if we define $\lambda: \cF \to [0,1]$ with We'll show that if we define $\lambda: \cF \to [0,1]$ with
$\lambda(A) = \lambda_n(A)$ for any $n$ where this is well defined, $\lambda(A) = \lambda_n(A)$ for any $n$ where this is well defined,
then $\lambda$ is countably additive on $\cF$. then $\lambda$ is countably additive on $\cF$.
Using \autoref{caratheodory}, $\lambda$ will extend uniquely to a probability measure on $\sigma(\cF)$. Using \yaref{caratheodory}, $\lambda$ will extend uniquely to a probability measure on $\sigma(\cF)$.
We want to prove: We want to prove:
\begin{claim} \begin{claim}
@ -107,7 +107,7 @@ We want to prove:
thus $\cF \subseteq \cB_\infty$. Since $\cB_\infty$ is a $\sigma$-algebra, thus $\cF \subseteq \cB_\infty$. Since $\cB_\infty$ is a $\sigma$-algebra,
$\sigma(\cF) \subseteq \cB_\infty$. $\sigma(\cF) \subseteq \cB_\infty$.
\end{refproof} \end{refproof}
For the proof of \autoref{claim:lambdacountadd}, For the proof of \yaref{claim:lambdacountadd},
we are going to use the following: we are going to use the following:
\begin{fact} \begin{fact}
\label{fact:finaddtocountadd} \label{fact:finaddtocountadd}

View File

@ -1,6 +1,6 @@
\lecture{4}{}{End of proof of Kolmogorov's consistency theorem} \lecture{4}{}{End of proof of Kolmogorov's consistency theorem}
To finish the proof of \autoref{claim:lambdacountadd}, To finish the proof of \yaref{claim:lambdacountadd},
we need the following: we need the following:
\begin{fact} \begin{fact}
\label{lec4fact1} \label{lec4fact1}
@ -91,7 +91,7 @@ we need the following:
so $\{x_k^{(n)}\}_n$ is bounded. so $\{x_k^{(n)}\}_n$ is bounded.
\end{itemize} \end{itemize}
By \autoref{lec4fact1}, By \yaref{lec4fact1},
there is an infinite set $S \subseteq \N$, there is an infinite set $S \subseteq \N$,
such that $\{x_k^{(n)}\}_{n \in S}$ such that $\{x_k^{(n)}\}_{n \in S}$
converges for every $k$. converges for every $k$.
@ -115,7 +115,7 @@ we need the following:
\end{refproof} \end{refproof}
\begin{refproof}{claim:lambdacountadd} \begin{refproof}{claim:lambdacountadd}
In order to apply \autoref{fact:finaddtocountadd}, In order to apply \yaref{fact:finaddtocountadd},
we need the following: we need the following:
\begin{claim} \begin{claim}
For any sequence $B_n \in \cF$ For any sequence $B_n \in \cF$
@ -163,7 +163,7 @@ we need the following:
\[ \[
\bigcap_{k=1}^n L_k^\ast \neq \emptyset. \bigcap_{k=1}^n L_k^\ast \neq \emptyset.
\] \]
By \autoref{lem:intersectioncompactsets}, By \yaref{lem:intersectioncompactsets},
it follows that it follows that
\[ \[
\bigcap_{k \in \N} L_k^\ast \neq \emptyset. \bigcap_{k \in \N} L_k^\ast \neq \emptyset.
@ -190,7 +190,7 @@ hence
\end{IEEEeqnarray*} \end{IEEEeqnarray*}
For the definition of $\lambda$ For the definition of $\lambda$
as well as the proof of \autoref{claim:lambdacountadd} as well as the proof of \yaref{claim:lambdacountadd}
we have only used that $(\lambda_n)_{n \in \N}$ we have only used that $(\lambda_n)_{n \in \N}$
is a consistent family. is a consistent family.
Hence we have in fact shown \autoref{thm:kolmogorovconsistency}. Hence we have in fact shown \yaref{thm:kolmogorovconsistency}.

View File

@ -9,7 +9,7 @@ The RHS is constant, which we can explicitly compute from the distribution of th
We fix a probability space $(\Omega, \cF, \bP)$ once and for all. We fix a probability space $(\Omega, \cF, \bP)$ once and for all.
\begin{theorem} \begin{theorem}
\label{lln} \label{lln} % TODO - yaref
Let $X_1, X_2,\ldots$ be i.i.d.~random variables on $(\R, \cB(\R))$ Let $X_1, X_2,\ldots$ be i.i.d.~random variables on $(\R, \cB(\R))$
and $m = \bE[X_i] < \infty$ and $m = \bE[X_i] < \infty$
and $\sigma^{2} = \Var(X_i) = \bE[ (X_i - \bE(X_i))^2] = \bE[X_i^2] - \bE[X_i]^2 < \infty$. and $\sigma^{2} = \Var(X_i) = \bE[ (X_i - \bE(X_i))^2] = \bE[X_i^2] - \bE[X_i]^2 < \infty$.
@ -37,7 +37,7 @@ We fix a probability space $(\Omega, \cF, \bP)$ once and for all.
\begin{IEEEeqnarray*}{rCl} \begin{IEEEeqnarray*}{rCl}
\bP\left[ \left| \frac{X_1 + \ldots + X_n}{n} - m\right| > \epsilon\right] \bP\left[ \left| \frac{X_1 + \ldots + X_n}{n} - m\right| > \epsilon\right]
&=& \bP\left[\left|\frac{S_n}{n}-m\right| > \epsilon\right]\\ &=& \bP\left[\left|\frac{S_n}{n}-m\right| > \epsilon\right]\\
&\overset{\text{Chebyshev}}{\le }& &\overset{\yaref{thm:chebyshev}}{\le }&
\frac{\Var\left( \frac{S_n}{n} \right) }{\epsilon^2} \frac{\Var\left( \frac{S_n}{n} \right) }{\epsilon^2}
= \frac{1}{n} \frac{\Var(X_1)}{\epsilon^2} = \frac{1}{n} \frac{\Var(X_1)}{\epsilon^2}
\xrightarrow{n \to \infty} 0 \xrightarrow{n \to \infty} 0
@ -69,7 +69,7 @@ Consider the following:
where $X_n$ has distribution where $X_n$ has distribution
$\frac{1}{n^2} \delta_n + \frac{1}{n^2} \delta_{-n} + (1-\frac{2}{n^2}) \delta_0$. $\frac{1}{n^2} \delta_n + \frac{1}{n^2} \delta_{-n} + (1-\frac{2}{n^2}) \delta_0$.
We have $\bP[X_n \neq 0] = \frac{2}{n^2}$. We have $\bP[X_n \neq 0] = \frac{2}{n^2}$.
Since this is summable, Borel-Cantelli yields Since this is summable, \yaref{thm:borelcantelli} yields
\[ \[
\bP[X_{n} \neq 0 \text{ for infinitely many $n$}] = 0. \bP[X_{n} \neq 0 \text{ for infinitely many $n$}] = 0.
\] \]

View File

@ -1,6 +1,6 @@
\lecture{6}{}{Proof of SLLN} \lecture{6}{}{Proof of SLLN}
\begin{refproof}{lln} \begin{refproof}{lln}
We want to deduce the SLLN (\autoref{lln}) from \autoref{thm2}. We want to deduce the SLLN (\yaref{lln}) from \yaref{thm2}.
W.l.o.g.~let us assume that $\bE[X_i] = 0$ W.l.o.g.~let us assume that $\bE[X_i] = 0$
(otherwise define $X'_i \coloneqq X_i - \bE[X_i]$). (otherwise define $X'_i \coloneqq X_i - \bE[X_i]$).
We will show that $\frac{S_n}{n} \xrightarrow{a.s.} 0$. We will show that $\frac{S_n}{n} \xrightarrow{a.s.} 0$.
@ -8,7 +8,7 @@
Then the $Y_i$ are independent and we have $\bE[Y_i] = 0$ Then the $Y_i$ are independent and we have $\bE[Y_i] = 0$
and $\Var(Y_i) = \frac{\sigma^2}{i^2}$. and $\Var(Y_i) = \frac{\sigma^2}{i^2}$.
Thus $\sum_{i=1}^\infty \Var(Y_i) < \infty$. Thus $\sum_{i=1}^\infty \Var(Y_i) < \infty$.
From \autoref{thm2} we obtain that $\sum_{i=1}^\infty Y_i$ converges a.s. From \yaref{thm2} we obtain that $\sum_{i=1}^\infty Y_i$ converges a.s.
\begin{claim} \begin{claim}
Let $(a_n)$ be a sequence in $\R$ Let $(a_n)$ be a sequence in $\R$
such that $\sum_{n=1}^{\infty} \frac{a_n}{n}$ converges, such that $\sum_{n=1}^{\infty} \frac{a_n}{n}$ converges,
@ -45,9 +45,9 @@
The SLLN follows from the claim. The SLLN follows from the claim.
\end{refproof} \end{refproof}
In order to prove \autoref{thm2}, we need the following: In order to prove \yaref{thm2}, we need the following:
\begin{theorem}[Kolmogorov's inequality] \begin{theorem}[Kolmogorov's inequality]
\label{thm:kolmogorovineq} \yalabel{Kolmogorov's Inequality}{Kolmogorov}{thm:kolmogorovineq}
If $X_1,\ldots, X_n$ are independent with $\bE[X_i] = 0$ If $X_1,\ldots, X_n$ are independent with $\bE[X_i] = 0$
and $\Var(X_i) = \sigma_i^2$, then and $\Var(X_i) = \sigma_i^2$, then
\[ \[
@ -139,7 +139,7 @@ In order to prove \autoref{thm2}, we need the following:
\begin{IEEEeqnarray*}{rCl} \begin{IEEEeqnarray*}{rCl}
&&\max \{|S_{m+1} - S_m|, |S_{m+2} - S_m|, \ldots, |S_{m+l} - S_m|\}\\ &&\max \{|S_{m+1} - S_m|, |S_{m+2} - S_m|, \ldots, |S_{m+l} - S_m|\}\\
&=& \max \{|X_{m+1}|, |X_{m+1} + X_{m+2}|, \ldots, |X_{m+1} + X_{m+2} + \ldots + X_{m+l}|\}\\ &=& \max \{|X_{m+1}|, |X_{m+1} + X_{m+2}|, \ldots, |X_{m+1} + X_{m+2} + \ldots + X_{m+l}|\}\\
&\overset{\text{\autoref{thm:kolmogorovineq}}}{\le}& &\overset{\yaref{thm:kolmogorovineq}}{\le}&
\frac{1}{\epsilon^2} \sum_{i=m}^{l} \Var(X_i)\\ \frac{1}{\epsilon^2} \sum_{i=m}^{l} \Var(X_i)\\
&\le & \frac{1}{\epsilon^2} \sum_{i=m}^\infty \Var(X_i) &\le & \frac{1}{\epsilon^2} \sum_{i=m}^\infty \Var(X_i)
\xrightarrow{m \to \infty} 0, \xrightarrow{m \to \infty} 0,
@ -200,6 +200,3 @@ In order to prove \autoref{thm2}, we need the following:
Hence $\frac{t}{N_t} \to m$. Hence $\frac{t}{N_t} \to m$.
\end{proof} \end{proof}

View File

@ -5,7 +5,7 @@
when the $X_n$ are independent. when the $X_n$ are independent.
\end{goal} \end{goal}
\begin{theorem}[Kolmogorov's three-series theorem] % Theorem 3 \begin{theorem}[Kolmogorov's three-series theorem] % Theorem 3
\label{thm:kolmogorovthreeseries} \yalabel{Kolmogorov's Three-Series Theorem}{3 Series}{thm:kolmogorovthreeseries}
\label{thm3} \label{thm3}
Let $X_n$ be a family of independent random variables. Let $X_n$ be a family of independent random variables.
\begin{enumerate}[(a)] \begin{enumerate}[(a)]
@ -21,7 +21,7 @@
Then all three series above converge for every $C > 0$. Then all three series above converge for every $C > 0$.
\end{enumerate} \end{enumerate}
\end{theorem} \end{theorem}
For the proof we'll need a slight generalization of \autoref{thm2}: For the proof we'll need a slight generalization of \yaref{thm2}:
\begin{theorem} %[Theorem 4] \begin{theorem} %[Theorem 4]
\label{thm4} \label{thm4}
Let $\{X_n\}_n$ be independent and \vocab{uniformly bounded} Let $\{X_n\}_n$ be independent and \vocab{uniformly bounded}
@ -31,7 +31,7 @@ For the proof we'll need a slight generalization of \autoref{thm2}:
converge. converge.
\end{theorem} \end{theorem}
\begin{refproof}{thm3} \begin{refproof}{thm3}
Assume, that we have already proved \autoref{thm4}. Assume, that we have already proved \yaref{thm4}.
We prove part (a) first. We prove part (a) first.
Put $Y_n = X_n \cdot \One_{\{|X_n| \le C\}}$. Put $Y_n = X_n \cdot \One_{\{|X_n| \le C\}}$.
Since the $X_n$ are independent, the $Y_n$ are independent as well. Since the $X_n$ are independent, the $Y_n$ are independent as well.
@ -40,11 +40,11 @@ For the proof we'll need a slight generalization of \autoref{thm2}:
$\sum_{n \ge 1} \int_{|X_n| \le C} X_n \dif\bP = \sum_{n \ge 1} \bE[Y_n]$ $\sum_{n \ge 1} \int_{|X_n| \le C} X_n \dif\bP = \sum_{n \ge 1} \bE[Y_n]$
and $\sum_{n \ge 1} \int_{|X_n| \le C} X_n^2 \dif\bP - \left( \int_{|X_n| \le C} X_n \dif\bP \right)^2 = \sum_{n \ge 1} \Var(Y_n)$ and $\sum_{n \ge 1} \int_{|X_n| \le C} X_n^2 \dif\bP - \left( \int_{|X_n| \le C} X_n \dif\bP \right)^2 = \sum_{n \ge 1} \Var(Y_n)$
converges. converges.
By \autoref{thm4} it follows that $\sum_{n \ge 1} Y_n < \infty$ By \yaref{thm4} it follows that $\sum_{n \ge 1} Y_n < \infty$
almost surely. almost surely.
Let $A_n \coloneqq \{\omega : |X_n(\omega)| > C\}$. Let $A_n \coloneqq \{\omega : |X_n(\omega)| > C\}$.
Since $\sum_{n \ge 1} \bP(A_n) < \infty$ by assumption, Since $\sum_{n \ge 1} \bP(A_n) < \infty$ by assumption,
Borel-Cantelli yields $\bP[\text{infinitely many $A_n$ occur}] = 0$. \yaref{thm:borelcantelli} yields $\bP[\text{infinitely many $A_n$ occur}] = 0$.
For the proof of (b), suppose $\sum_{n\ge 1} X_n(\omega) < \infty$ For the proof of (b), suppose $\sum_{n\ge 1} X_n(\omega) < \infty$
@ -59,7 +59,7 @@ For the proof we'll need a slight generalization of \autoref{thm2}:
\] \]
Then the $Y_n$ are independent and $\sum_{n \ge 1} Y_n(\omega) < \infty$ Then the $Y_n$ are independent and $\sum_{n \ge 1} Y_n(\omega) < \infty$
almost surely and the $Y_n$ are uniformly bounded. almost surely and the $Y_n$ are uniformly bounded.
By \autoref{thm4} $\sum_{n \ge 1} \bE[Y_n]$ and $\sum_{n \ge 1} \Var(Y_n)$ By \yaref{thm4} $\sum_{n \ge 1} \bE[Y_n]$ and $\sum_{n \ge 1} \Var(Y_n)$
converge. converge.
Define Define
\[ \[
@ -70,7 +70,7 @@ For the proof we'll need a slight generalization of \autoref{thm2}:
\] \]
Then the $Z_n$ are independent, uniformly bounded and $\sum_{n \ge 1} Z_n(\omega) < \infty$ Then the $Z_n$ are independent, uniformly bounded and $\sum_{n \ge 1} Z_n(\omega) < \infty$
almost surely. almost surely.
By \autoref{thm4} we have By \yaref{thm4} we have
$\sum_{n \ge 1} \bE(Z_n) < \infty$ $\sum_{n \ge 1} \bE(Z_n) < \infty$
and $\sum_{n \ge 1} \Var(Z_n) < \infty$. and $\sum_{n \ge 1} \Var(Z_n) < \infty$.
@ -88,8 +88,8 @@ For the proof we'll need a slight generalization of \autoref{thm2}:
$\sum_{n \ge 1} \Var(Z_n)$ to conclude that this series converges $\sum_{n \ge 1} \Var(Z_n)$ to conclude that this series converges
as well. as well.
\end{refproof} \end{refproof}
Recall \autoref{thm2}. Recall \yaref{thm2}.
We will see, that the converse of \autoref{thm2} is true if the $X_n$ are uniformly bounded. We will see, that the converse of \yaref{thm2} is true if the $X_n$ are uniformly bounded.
More formally: More formally:
\begin{theorem}[Theorem 5] \begin{theorem}[Theorem 5]
\label{thm5} \label{thm5}
@ -99,14 +99,14 @@ More formally:
then $\sum_{n \ge 1} \Var(X_n) < \infty$. then $\sum_{n \ge 1} \Var(X_n) < \infty$.
\end{theorem} \end{theorem}
\begin{refproof}{thm4} \begin{refproof}{thm4}
Assume we have proven \autoref{thm5}. Assume we have proven \yaref{thm5}.
``$\impliedby$'' Assume $\{X_n\} $ are independent, uniformly bounded ``$\impliedby$'' Assume $\{X_n\} $ are independent, uniformly bounded
and $\sum_{n \ge 1} \bE(X_n) < \infty$ as well as $\sum_{n \ge 1} \Var(X_n) < \infty$. and $\sum_{n \ge 1} \bE(X_n) < \infty$ as well as $\sum_{n \ge 1} \Var(X_n) < \infty$.
We need to show that $\sum_{n \ge 1} X_n < \infty$ a.s. We need to show that $\sum_{n \ge 1} X_n < \infty$ a.s.
Let $Y_n \coloneqq X_n - \bE(X_n)$. Let $Y_n \coloneqq X_n - \bE(X_n)$.
Then the $Y_n$ are independent, $\bE(Y_n) = 0$ and $\Var(Y_n) = \Var(X_n)$. Then the $Y_n$ are independent, $\bE(Y_n) = 0$ and $\Var(Y_n) = \Var(X_n)$.
By \autoref{thm2} $\sum_{n \ge 1} Y_n < \infty$ a.s. By \yaref{thm2} $\sum_{n \ge 1} Y_n < \infty$ a.s.
Thus $\sum_{n \ge 1} X_n < \infty$ a.s. Thus $\sum_{n \ge 1} X_n < \infty$ a.s.
``$\implies$'' We assume that $\{X_n\}$ are independent, uniformly bounded ``$\implies$'' We assume that $\{X_n\}$ are independent, uniformly bounded
@ -145,23 +145,23 @@ More formally:
$\sum_{n \ge 1} \left(Y_n(\omega, \omega') - Z_n(\omega, \omega') \right)= \sum_{n \ge 1} \left(X_n(\omega) - X_n(\omega')\right)$. $\sum_{n \ge 1} \left(Y_n(\omega, \omega') - Z_n(\omega, \omega') \right)= \sum_{n \ge 1} \left(X_n(\omega) - X_n(\omega')\right)$.
Thus $\sum_{n \ge 1} \left( Y_n(\omega, \omega') - Z_n(\omega, \omega') \right) < \infty$ a.s.~on $\Omega_0\otimes\Omega_0$. Thus $\sum_{n \ge 1} \left( Y_n(\omega, \omega') - Z_n(\omega, \omega') \right) < \infty$ a.s.~on $\Omega_0\otimes\Omega_0$.
\end{subproof} \end{subproof}
By \autoref{thm5}, $\sum_{n} \Var(X_n) = \frac{1}{2}\sum_{n \ge 1} \Var(Y_n - Z_n) < \infty$ a.s. By \yaref{thm5}, $\sum_{n} \Var(X_n) = \frac{1}{2}\sum_{n \ge 1} \Var(Y_n - Z_n) < \infty$ a.s.
Define $U_n \coloneqq X_n - \bE(X_n)$. Define $U_n \coloneqq X_n - \bE(X_n)$.
Then $\bE(U_n) = 0$ and the $U_n$ are independent Then $\bE(U_n) = 0$ and the $U_n$ are independent
and uniformly bounded. and uniformly bounded.
We have $\sum_{n} \Var(U_n) = \sum_{n} \Var(X_n) < \infty$. We have $\sum_{n} \Var(U_n) = \sum_{n} \Var(X_n) < \infty$.
Thus $\sum_{n} U_n$ converges a.s.~by \autoref{thm2}. Thus $\sum_{n} U_n$ converges a.s.~by \yaref{thm2}.
Since by assumption $\sum_{n} X_n < \infty$ a.s., Since by assumption $\sum_{n} X_n < \infty$ a.s.,
it follows that $\sum_{n} \bE(X_n) < \infty$. it follows that $\sum_{n} \bE(X_n) < \infty$.
\end{refproof} \end{refproof}
\begin{remark} \begin{remark}
In the proof of \autoref{thm4} In the proof of \yaref{thm4}
``$\impliedby$'' is just a trivial application of \autoref{thm2} ``$\impliedby$'' is just a trivial application of \yaref{thm2}
and uniform boundedness was not used. and uniform boundedness was not used.
The idea of `` $\implies$ '' will lead to coupling. % TODO ? The idea of `` $\implies$ '' will lead to coupling. % TODO ?
\end{remark} \end{remark}
A proof of \autoref{thm5} can be found in the notes.\notes A proof of \yaref{thm5} can be found in the notes.\notes
\begin{example}[Application of \autoref{thm4}] \begin{example}[Application of \yaref{thm4}]
The series $\sum_{n} \frac{1}{n^{\frac{1}{2} + \epsilon}}$ The series $\sum_{n} \frac{1}{n^{\frac{1}{2} + \epsilon}}$
does not converge for $\epsilon < \frac{1}{2}$. does not converge for $\epsilon < \frac{1}{2}$.
However However

View File

@ -59,7 +59,7 @@ which does not depend on the realisation of the first $k$ random variables
for any $k \in \N$. for any $k \in \N$.
\begin{theorem}[Kolmogorov's 0-1 law] \begin{theorem}[Kolmogorov's 0-1 law]
\label{kolmogorov01} \yalabel{Kolmogorov's 0-1 Law}{0-1 Law}{kolmogorov01}
Let $X_n, n \in \N$ be a sequence of independent random variables Let $X_n, n \in \N$ be a sequence of independent random variables
and let $\cT$ denote their tail-$\sigma$-algebra. and let $\cT$ denote their tail-$\sigma$-algebra.
Then $\cT$ is \vocab{$\bP$-trivial}, i.e.~$\bP[A] \in \{0,1\}$ Then $\cT$ is \vocab{$\bP$-trivial}, i.e.~$\bP[A] \in \{0,1\}$

View File

@ -1,7 +1,7 @@
\lecture{9}{}{Percolation, Introduction to characteristic functions} \lecture{9}{}{Percolation, Introduction to characteristic functions}
\subsubsection{Application: Percolation} \subsubsection{Application: Percolation}
We will now discuss another application of Kolmogorov's $0-1$-law, percolation. We will now discuss another application of \yaref{kolmogorov01}, percolation.
\begin{definition}[\vocab{Percolation}] \begin{definition}[\vocab{Percolation}]
Consider the graph with nodes $\Z^d$, $d \ge 2$, where edges from the lattice are added with probability $p$. The added edges are called \vocab[Percolation!Edge!open]{open}; Consider the graph with nodes $\Z^d$, $d \ge 2$, where edges from the lattice are added with probability $p$. The added edges are called \vocab[Percolation!Edge!open]{open};
@ -178,7 +178,7 @@ We have
\end{remark} \end{remark}
\begin{theorem}[Inversion formula] % thm1 \begin{theorem}[Inversion formula] % thm1
\label{inversionformula} \yalabel{Inversion Formula}{Inversion Formula}{inversionformula}
Let $(\Omega, \cB(\R), \bP)$ be a probability space. Let $(\Omega, \cB(\R), \bP)$ be a probability space.
Let $F$ be the distribution function of $\bP$ Let $F$ be the distribution function of $\bP$
(i.e.~$F(x) = \bP((-\infty, x])$ for all $x \in \R$ ). (i.e.~$F(x) = \bP((-\infty, x])$ for all $x \in \R$ ).
@ -193,7 +193,7 @@ We have
We will prove this later. We will prove this later.
\begin{theorem}[Uniqueness theorem] % thm2 \begin{theorem}[Uniqueness theorem] % thm2
\label{charfuncuniqueness} \yalabel{Uniqueness Theorem}{Uniqueness}{charfuncuniqueness}
Let $\bP$ and $\Q$ be two probability measures on $(\R, \cB(\R))$. Let $\bP$ and $\Q$ be two probability measures on $(\R, \cB(\R))$.
Then $\phi_\bP = \phi_\Q \implies \bP = \Q$. Then $\phi_\bP = \phi_\Q \implies \bP = \Q$.
@ -202,20 +202,20 @@ We will prove this later.
from $\phi$. from $\phi$.
\end{theorem} \end{theorem}
\begin{refproof}{charfuncuniqueness} \begin{refproof}{charfuncuniqueness}
Assume that we have already shown \autoref{inversionformula}. Assume that we have already shown the \yaref{inversionformula}.
Suppose that $F$ and $G$ are the distribution functions of $\bP$ and $\Q$. Suppose that $F$ and $G$ are the distribution functions of $\bP$ and $\Q$.
Let $a,b \in \R$ with $a < b$. Let $a,b \in \R$ with $a < b$.
Assume that $a $ and $b$ are continuity points of both $F$ and $G$. Assume that $a $ and $b$ are continuity points of both $F$ and $G$.
By \autoref{inversionformula} we have By the \yaref{inversionformula} we have
\begin{IEEEeqnarray*}{rCl} \begin{IEEEeqnarray*}{rCl}
F(b) - F(a) = G(b) - G(a) \label{eq:charfuncuniquefg} F(b) - F(a) = G(b) - G(a) \label{eq:charfuncuniquefg}
\end{IEEEeqnarray*} \end{IEEEeqnarray*}
Since $F$ and $G$ are monotonic, \autoref{eq:charfuncuniquefg} Since $F$ and $G$ are monotonic, \yaref{eq:charfuncuniquefg}
holds for all $a < b$ outside a countable set. holds for all $a < b$ outside a countable set.
Take $a_n$ outside this countable set, such that $a_n \ssearrow -\infty$. Take $a_n$ outside this countable set, such that $a_n \ssearrow -\infty$.
Then, \autoref{eq:charfuncuniquefg} implies that Then, \yaref{eq:charfuncuniquefg} implies that
$F(b) - F(a_n) = G(b) - G(a_n)$ hence $F(b) = G(b)$. $F(b) - F(a_n) = G(b) - G(a_n)$ hence $F(b) = G(b)$.
Since $F$ and $G$ are right-continuous, it follows that $F = G$. Since $F$ and $G$ are right-continuous, it follows that $F = G$.
\end{refproof} \end{refproof}

View File

@ -14,7 +14,7 @@ where $\mu = \bP X^{-1}$.
\begin{refproof}{inversionformula} \begin{refproof}{inversionformula}
We will prove that the limit in the RHS of \autoref{invf} We will prove that the limit in the RHS of \yaref{invf}
exists and is equal to the LHS. exists and is equal to the LHS.
Note that the term on the RHS is integrable, as Note that the term on the RHS is integrable, as
\[ \[
@ -31,7 +31,7 @@ where $\mu = \bP X^{-1}$.
&=& \lim_{T \to \infty} \frac{1}{2 \pi} \int_{\R} \underbrace{\int_{-T}^T \left[ \frac{\cos(t (x-b)) - \cos(t(x-a))}{-\i t}\right] \dif t}_{=0 \text{, as the function is odd}} \bP(\dif x) \\ &=& \lim_{T \to \infty} \frac{1}{2 \pi} \int_{\R} \underbrace{\int_{-T}^T \left[ \frac{\cos(t (x-b)) - \cos(t(x-a))}{-\i t}\right] \dif t}_{=0 \text{, as the function is odd}} \bP(\dif x) \\
&& + \lim_{T \to \infty} \frac{1}{2\pi} \int_{\R}\int_{-T}^T \frac{\sin(t ( x - b)) - \sin(t(x-a))}{-t} \dif t \bP(\dif x)\\ && + \lim_{T \to \infty} \frac{1}{2\pi} \int_{\R}\int_{-T}^T \frac{\sin(t ( x - b)) - \sin(t(x-a))}{-t} \dif t \bP(\dif x)\\
&=& \lim_{T \to \infty} \frac{1}{\pi} \int_\R \int_{0}^T \frac{\sin(t(x-a)) - \sin(t(x-b))}{t} \dif t \bP(\dif x)\\ &=& \lim_{T \to \infty} \frac{1}{\pi} \int_\R \int_{0}^T \frac{\sin(t(x-a)) - \sin(t(x-b))}{t} \dif t \bP(\dif x)\\
&\overset{\substack{\text{\autoref{fact:sincint},}\\\text{dominated convergence}}}{=}& &\overset{\substack{\yaref{fact:sincint},\text{dominated convergence}}}{=}&
\frac{1}{\pi} \int -\frac{\pi}{2} \One_{x < a} + \frac{\pi}{2} \One_{x > a} \frac{1}{\pi} \int -\frac{\pi}{2} \One_{x < a} + \frac{\pi}{2} \One_{x > a}
- (- \frac{\pi}{2} \One_{x < b} + \frac{\pi}{2} \One_{x > b}) \bP(\dif x)\\ - (- \frac{\pi}{2} \One_{x < b} + \frac{\pi}{2} \One_{x > b}) \bP(\dif x)\\
&=& \frac{1}{2} \bP(\{a\} ) + \frac{1}{2} \bP(\{b\}) + \bP((a,b))\\ &=& \frac{1}{2} \bP(\{a\} ) + \frac{1}{2} \bP(\{b\}) + \bP((a,b))\\
@ -103,7 +103,7 @@ where $\mu = \bP X^{-1}$.
\bP\left( (a,b] \right) = \int_a^b f(x) \dif x.\label{thm10_3eq1} \bP\left( (a,b] \right) = \int_a^b f(x) \dif x.\label{thm10_3eq1}
\] \]
Let $F$ be the distribution function of $\bP$. Let $F$ be the distribution function of $\bP$.
It is enough to prove \autoref{thm10_3eq1} It is enough to prove \yaref{thm10_3eq1}
for all continuity points $a $ and $ b$ of $F$. for all continuity points $a $ and $ b$ of $F$.
We have We have
\begin{IEEEeqnarray*}{rCl} \begin{IEEEeqnarray*}{rCl}
@ -112,12 +112,14 @@ where $\mu = \bP X^{-1}$.
&=& \frac{1}{2\pi} \int_{\R} \phi(t) \left( \frac{e^{-\i t b} - e^{-\i t a}}{- \i t} \right) \dif t\\ &=& \frac{1}{2\pi} \int_{\R} \phi(t) \left( \frac{e^{-\i t b} - e^{-\i t a}}{- \i t} \right) \dif t\\
&\overset{\text{dominated convergence}}{=}& \lim_{T \to \infty} \frac{1}{2\pi} \int_{-T}^{T} \phi(t) \left( \frac{e^{-\i t b} - e^{- \i t a}}{- \i t} \right) \dif t &\overset{\text{dominated convergence}}{=}& \lim_{T \to \infty} \frac{1}{2\pi} \int_{-T}^{T} \phi(t) \left( \frac{e^{-\i t b} - e^{- \i t a}}{- \i t} \right) \dif t
\end{IEEEeqnarray*} \end{IEEEeqnarray*}
By \autoref{inversionformula}, the RHS is equal to $F(b) - F(a) = \bP\left( (a,b] \right)$. By the \yaref{inversionformula},
the RHS is equal to $F(b) - F(a) = \bP\left( (a,b] \right)$.
\end{refproof} \end{refproof}
However, Fourier analysis is not only useful for continuous probability density functions: However, Fourier analysis is not only useful for continuous probability density functions:
\begin{theorem}[Bochner's formula for the mass at a point]\label{bochnersformula} % Theorem 4 \begin{theorem}[Bochner's formula for the mass at a point]
\yalabel{Bochner's Formula for the Mass at a Point}{Bochner}{bochnersformula} % Theorem 4
Let $\bP \in M_1(\lambda)$. Let $\bP \in M_1(\lambda)$.
Then Then
\[ \[
@ -174,13 +176,14 @@ However, Fourier analysis is not only useful for continuous probability density
&=& \int_{\R} \left| \sum_{l} c_l e^{\i t_l x}\right|^2 \ge 0 &=& \int_{\R} \left| \sum_{l} c_l e^{\i t_l x}\right|^2 \ge 0
\end{IEEEeqnarray*} \end{IEEEeqnarray*}
\end{refproof} \end{refproof}
\begin{theorem}[Bochner's theorem]\label{thm:bochner} \begin{theorem}[Bochner's theorem]
The converse to \autoref{thm:lec_10thm5} holds, i.e.~any \yalabel{Bochner's Theorem for Positive Definite Functions}{Bochner's Theorem}{thm:bochner}%
$\phi: \R \to \C$ satisfying (a) and (b) of \autoref{thm:lec_10thm5} The converse to \yaref{thm:lec_10thm5} holds, i.e.~any
$\phi: \R \to \C$ satisfying (a) and (b) of \yaref{thm:lec_10thm5}
must be the Fourier transform of a probability measure $\bP$ must be the Fourier transform of a probability measure $\bP$
on $(\R, \cB(\R))$. on $(\R, \cB(\R))$.
\end{theorem} \end{theorem}
Unfortunately, we won't prove \autoref{thm:bochner} in this lecture. Unfortunately, we won't prove \yaref{thm:bochner} in this lecture.
\begin{definition}[Convergence in distribution / weak convergence] \begin{definition}[Convergence in distribution / weak convergence]
@ -325,7 +328,8 @@ for all $f \in C_b(\R)$.
% \end{itemize} % \end{itemize}
% %
% \end{proof} % \end{proof}
\begin{theorem}[Levy's continuity theorem]\label{levycontinuity} \begin{theorem}[Levy's continuity theorem]
\yalabel{Levy's Continuity Theorem}{Levy}{levycontinuity}
% Theorem 2 % Theorem 2
$X_n \xrightarrow{\text{d}} X$ iff $X_n \xrightarrow{\text{d}} X$ iff
$\phi_{X_n}(t) \to \phi(t)$ for all $t \in \R$. $\phi_{X_n}(t) \to \phi(t)$ for all $t \in \R$.

View File

@ -52,7 +52,8 @@ In order to make things nicer, we do the following:
Then $\bE[\frac{S_n - \bE[S_n]}{\sqrt{\Var(S_n)}}] = 0$ Then $\bE[\frac{S_n - \bE[S_n]}{\sqrt{\Var(S_n)}}] = 0$
and $\Var(\frac{S_n - \bE[S_n]}{\sqrt{\Var(S_n)}}) = 1$. and $\Var(\frac{S_n - \bE[S_n]}{\sqrt{\Var(S_n)}}) = 1$.
\begin{theorem}[Central limit theorem, 1920s, Lindeberg and Levy]\label{clt} \begin{theorem}[Central limit theorem, 1920s, Lindeberg and Levy]%
\yalabel{Central Limit Theorem}{CLT}{clt}
Let $X_1,X_2,\ldots$ be i.i.d.~random variables Let $X_1,X_2,\ldots$ be i.i.d.~random variables
with $\bE[X_1] = \mu$ and $\Var(X_1) = \sigma^2 \in (0, \infty)$. with $\bE[X_1] = \mu$ and $\Var(X_1) = \sigma^2 \in (0, \infty)$.
@ -81,9 +82,9 @@ There exists a special case of this theorem, which was proved earlier:
Let $X_1, X_2,\ldots$ i.i.d.~with $X_1 \sim \Ber(p)$. Let $X_1, X_2,\ldots$ i.i.d.~with $X_1 \sim \Ber(p)$.
Then $\bE[X_1] = p$ and $\Var(X_1) = p(1-p )$. Then $\bE[X_1] = p$ and $\Var(X_1) = p(1-p )$.
Furthermore $\sum_{i=1}^n X_i \sim \Bin(n,p)$, Furthermore $\sum_{i=1}^n X_i \sim \Bin(n,p)$,
and the special case follows from \autoref{clt}. and the special case follows from \yaref{clt}.
\end{proof} \end{proof}
\autoref{preclt} is a useful tool for approximating the Binomial distribution with the normal distribution. \yaref{preclt} is a useful tool for approximating the Binomial distribution with the normal distribution.
If $S_n \sim \Bin(n,p)$ and $[a,b] \subseteq \R$, we have If $S_n \sim \Bin(n,p)$ and $[a,b] \subseteq \R$, we have
\[\bP[a \le S_n \le b] = \bP\left[\frac{a - np}{\sqrt{np(1-p)}} \le \frac{S_n -np}{\sqrt{n p (1-p)}} \le \frac{b - np}{\sqrt{n p (1-p)} }\right] \approx \Phi(b') - \Phi(a').\] \[\bP[a \le S_n \le b] = \bP\left[\frac{a - np}{\sqrt{np(1-p)}} \le \frac{S_n -np}{\sqrt{n p (1-p)}} \le \frac{b - np}{\sqrt{n p (1-p)} }\right] \approx \Phi(b') - \Phi(a').\]
@ -105,7 +106,7 @@ If $S_n \sim \Bin(n,p)$ and $[a,b] \subseteq \R$, we have
More formally: Let $X_1,X_2,\ldots$ be i.i.d.~with $\bP[X_1=1] = \bP[X_1=-1] = \frac{1}{2}$ and consider $S_n \coloneqq \sum_{i=1}^n X_i$. More formally: Let $X_1,X_2,\ldots$ be i.i.d.~with $\bP[X_1=1] = \bP[X_1=-1] = \frac{1}{2}$ and consider $S_n \coloneqq \sum_{i=1}^n X_i$.
Then \autoref{clt} states, that $S_n \approx \cN(0,n)$. Then the \yaref{clt} states, that $S_n \approx \cN(0,n)$.
\end{example} \end{example}
\begin{example} \begin{example}
@ -129,4 +130,3 @@ If $S_n \sim \Bin(n,p)$ and $[a,b] \subseteq \R$, we have
We have $p\cdot (1-p) \le \frac{1}{4}$, We have $p\cdot (1-p) \le \frac{1}{4}$,
thus $n \approx (1.96)^2 \cdot 100^2 \cdot \frac{1}{4} = 9600$ suffices. thus $n \approx (1.96)^2 \cdot 100^2 \cdot \frac{1}{4} = 9600$ suffices.
\end{example} \end{example}

View File

@ -1,12 +1,12 @@
\lecture{12}{2023-05-16}{Proof of the CLT} \lecture{12}{2023-05-16}{Proof of the CLT}
We now want to prove \autoref{clt}. We now want to prove the \yaref{clt}.
The plan is to do the following: The plan is to do the following:
\begin{enumerate}[1.] \begin{enumerate}[1.]
\item Identify the characteristic function of a standard normal \item Identify the characteristic function of a standard normal
\item Show that the characteristic functions of the $V_n$ converge pointwise \item Show that the characteristic functions of the $V_n$ converge pointwise
to that of $\cN$. to that of $\cN$.
\item Apply \autoref{levycontinuity} \item Apply \yaref{levycontinuity}
\end{enumerate} \end{enumerate}
First, we need to prove some properties of characteristic functions. First, we need to prove some properties of characteristic functions.
@ -94,7 +94,7 @@ First, we need to prove some properties of characteristic functions.
For arbitrary $h \in \R$, we have For arbitrary $h \in \R$, we have
\begin{IEEEeqnarray*}{rCl} \begin{IEEEeqnarray*}{rCl}
|e^{\i t X} \frac{e^{\i h X}}{h}| &\le & \left| \frac{1}{h} \left( e^{\i h X} - 1 \right)\right|\\ |e^{\i t X} \frac{e^{\i h X}}{h}| &\le & \left| \frac{1}{h} \left( e^{\i h X} - 1 \right)\right|\\
&\overset{\text{\autoref{charfprop:c1}}}{\le}& \left|\frac{1}{h} \i h X\right| = |X|. &\overset{\yaref{charfprop:c1}}{\le}& \left|\frac{1}{h} \i h X\right| = |X|.
\end{IEEEeqnarray*} \end{IEEEeqnarray*}
Thus the dominated convergence theorem can be applied and we obtain Thus the dominated convergence theorem can be applied and we obtain
\[ \[
@ -156,7 +156,7 @@ First, we need to prove some properties of characteristic functions.
\end{refproof} \end{refproof}
Now, we can finally prove the CLT: Now, we can finally prove the \yaref{clt}:
\begin{refproof}{clt} \begin{refproof}{clt}
Let $X_1,X_2,\ldots$ be i.i.d.~random variables Let $X_1,X_2,\ldots$ be i.i.d.~random variables
with $\bE[X_1] = \mu_1$, $\Var(X_1) = \sigma^2$. with $\bE[X_1] = \mu_1$, $\Var(X_1) = \sigma^2$.
@ -212,7 +212,7 @@ Now, we can finally prove the CLT:
\[ \[
\phi_n(t) \xrightarrow{n \to \infty} e^{-\frac{t^2}{2}} = \phi_{\cN(0,1)}(t). \phi_n(t) \xrightarrow{n \to \infty} e^{-\frac{t^2}{2}} = \phi_{\cN(0,1)}(t).
\] \]
Using \autoref{levycontinuity}, we obtain \autoref{clt}. Using \yaref{levycontinuity}, we obtain the \yaref{clt}.
\end{refproof} \end{refproof}
\begin{remark} \begin{remark}

View File

@ -1,5 +1,5 @@
\lecture{13}{2023-05}{} \lecture{13}{2023-05}{}
%The difficult part is to show \autoref{levycontinuity}. %The difficult part is to show \yaref{levycontinuity}.
%This is the last lecture, where we will deal with independent random variables. %This is the last lecture, where we will deal with independent random variables.
We have seen, that We have seen, that
@ -12,7 +12,7 @@ if $X_1, X_2,\ldots$ are i.i.d.~with $ \mu = \bE[X_1]$,
\end{question} \end{question}
\begin{theorem}[Lindeberg CLT] \begin{theorem}[Lindeberg CLT]
\label{lindebergclt} \yalabel{Lindeberg's CLT}{Lindeberg CLT}{lindebergclt}
Assume $X_1, X_2, \ldots,$ are independent (but not necessarily identically distributed) with $\mu_i = \bE[X_i] < \infty$ and $\sigma_i^2 = \Var(X_i) < \infty$. Assume $X_1, X_2, \ldots,$ are independent (but not necessarily identically distributed) with $\mu_i = \bE[X_i] < \infty$ and $\sigma_i^2 = \Var(X_i) < \infty$.
Let $S_n = \sqrt{\sum_{i=1}^{n} \sigma_i^2}$ Let $S_n = \sqrt{\sum_{i=1}^{n} \sigma_i^2}$
and assume that and assume that
@ -29,7 +29,7 @@ if $X_1, X_2,\ldots$ are i.i.d.~with $ \mu = \bE[X_1]$,
\end{theorem} \end{theorem}
\begin{theorem}[Lyapunov condition] \begin{theorem}[Lyapunov condition]
\label{lyapunovclt} \yalabel{Lyapunov's CLT}{Lyapunov CLT}{lyapunovclt}
Let $X_1, X_2,\ldots$ be independent, $\mu_i = \bE[X_i] < \infty$, Let $X_1, X_2,\ldots$ be independent, $\mu_i = \bE[X_i] < \infty$,
$\sigma_i^2 = \Var(X_i) < \infty$ $\sigma_i^2 = \Var(X_i) < \infty$
and $S_n \coloneqq \sqrt{\sum_{i=1}^n \sigma_i^2}$. and $S_n \coloneqq \sqrt{\sum_{i=1}^n \sigma_i^2}$.
@ -45,10 +45,10 @@ if $X_1, X_2,\ldots$ are i.i.d.~with $ \mu = \bE[X_1]$,
The Lyapunov condition implies the Lindeberg condition. The Lyapunov condition implies the Lindeberg condition.
(Exercise). (Exercise).
\end{remark} \end{remark}
We will not prove the \autoref{lindebergclt} or \autoref{lyapunovclt} We will not prove \yaref{lindebergclt} or \yaref{lyapunovclt}
in this lecture. However, they are quite important. in this lecture. However, they are quite important.
We will now sketch the proof of \autoref{levycontinuity}, We will now sketch the proof of \yaref{levycontinuity},
details can be found in the notes.\notes details can be found in the notes.\notes
\begin{definition} \begin{definition}
Let $(X_n)_n$ be a sequence of random variables. Let $(X_n)_n$ be a sequence of random variables.
@ -62,8 +62,8 @@ details can be found in the notes.\notes
\begin{example}+[Exercise 8.1] \begin{example}+[Exercise 8.1]
\todo{Copy} \todo{Copy}
\end{example} \end{example}
A generalized version of \autoref{levycontinuity} is the following: A generalized version of \yaref{levycontinuity} is the following:
\begin{theorem}[A generalized version of Levy's continuity \autoref{levycontinuity}] \begin{theorem}[A generalized version of \yaref{levycontinuity}]
\label{genlevycontinuity} \label{genlevycontinuity}
Suppose we have random variables $(X_n)_n$ such that Suppose we have random variables $(X_n)_n$ such that
$\bE[e^{\i t X_n}] \xrightarrow{n \to \infty} \phi(t)$ for all $t \in \R$ $\bE[e^{\i t X_n}] \xrightarrow{n \to \infty} \phi(t)$ for all $t \in \R$
@ -77,12 +77,12 @@ A generalized version of \autoref{levycontinuity} is the following:
\item $\phi$ is continuous at $0$. \item $\phi$ is continuous at $0$.
\end{enumerate} \end{enumerate}
\end{theorem} \end{theorem}
\todo{Proof of \autoref{genlevycontinuity} (Exercise 8.2)} \todo{Proof of \yaref{genlevycontinuity} (Exercise 8.2)}
\begin{example} \begin{example}
Let $Z \sim \cN(0,1)$ and $X_n \coloneqq n Z$. Let $Z \sim \cN(0,1)$ and $X_n \coloneqq n Z$.
We have $\phi_{X_n}(t) = \bE[[e^{\i t X_n}] = e^{-\frac{1}{2} t^2 n^2} \xrightarrow{n \to \infty} \One_{\{t = 0\} }$. We have $\phi_{X_n}(t) = \bE[[e^{\i t X_n}] = e^{-\frac{1}{2} t^2 n^2} \xrightarrow{n \to \infty} \One_{\{t = 0\} }$.
$\One_{\{t = 0\}}$ is not continuous at $0$. $\One_{\{t = 0\}}$ is not continuous at $0$.
By \autoref{genlevycontinuity}, $X_n$ can not converge to a real-valued By \yaref{genlevycontinuity}, $X_n$ can not converge to a real-valued
random variable. random variable.
Exercise: $X_n \xrightarrow{(d)} \overline{X}$, Exercise: $X_n \xrightarrow{(d)} \overline{X}$,
@ -102,12 +102,12 @@ A generalized version of \autoref{levycontinuity} is the following:
\frac{1}{n} \bE[ (X_1+ \ldots + X_n)^2]\\ \frac{1}{n} \bE[ (X_1+ \ldots + X_n)^2]\\
&=& \sigma^2 &=& \sigma^2
\end{IEEEeqnarray*} \end{IEEEeqnarray*}
For $a > 0$, by Chebyshev's inequality, % TODO For $a > 0$, by \yaref{thm:chebyshev},
we have we have
\[ \[
\bP\left[ \left| \frac{S_n}{\sqrt{n}} \right| > a \right] \leq \frac{\sigma^2}{a^2} \xrightarrow{a \to \infty} 0. \bP\left[ \left| \frac{S_n}{\sqrt{n}} \right| > a \right] \leq \frac{\sigma^2}{a^2} \xrightarrow{a \to \infty} 0.
\] \]
verifying \autoref{genlevycontinuity}. verifying \yaref{genlevycontinuity}.
\end{example} \end{example}
\begin{example} \begin{example}
@ -133,9 +133,9 @@ A generalized version of \autoref{levycontinuity} is the following:
Exercise: $\phi_{\frac{S_n}{n}}(t) = e^{-|t|} = \phi_{C_1}(t)$, thus $\frac{S_n}{n} \sim C$. Exercise: $\phi_{\frac{S_n}{n}}(t) = e^{-|t|} = \phi_{C_1}(t)$, thus $\frac{S_n}{n} \sim C$.
\end{example} \end{example}
We will prove \autoref{levycontinuity} assuming We will prove \yaref{levycontinuity} assuming
\autoref{lec10_thm1}. \yaref{lec10_thm1}.
\autoref{lec10_thm1} will be shown in the notes.\notes \yaref{lec10_thm1} will be shown in the notes.\notes
We will need the following: We will need the following:
\begin{lemma} \begin{lemma}
\label{lec13_lem1} \label{lec13_lem1}
@ -217,7 +217,7 @@ for all $t \in \R$.
Apply dominated convergence. Apply dominated convergence.
\end{subproof} \end{subproof}
So to prove $\mu_n\left( (-A,A) \right) \ge 1 - 2 \epsilon$, So to prove $\mu_n\left( (-A,A) \right) \ge 1 - 2 \epsilon$,
apply \autoref{s7e1}. apply \yaref{s7e1}.
It suffices to show that It suffices to show that
\[ \[
\frac{A}{2} \left| \int_{-\frac{2}{A}}^{\frac{2}{A}} \phi_n(t) dt\right| - 1 \ge 1 - 2\epsilon \frac{A}{2} \left| \int_{-\frac{2}{A}}^{\frac{2}{A}} \phi_n(t) dt\right| - 1 \ge 1 - 2\epsilon
@ -226,11 +226,11 @@ for all $t \in \R$.
\[ \[
1 - \frac{A}{4} \left|\int_{-\frac{2}{A}}^{\frac{2}{A}} \phi_n(t) dt \right| \le \epsilon, 1 - \frac{A}{4} \left|\int_{-\frac{2}{A}}^{\frac{2}{A}} \phi_n(t) dt \right| \le \epsilon,
\] \]
which follows from \autoref{levyproofc1eqn2}. which follows from \yaref{levyproofc1eqn2}.
\end{refproof} \end{refproof}
% Step 2 % Step 2
By \autoref{lec13_lem1} By \yaref{lec13_lem1}
there exists a right continuous, non-decreasing $F $ there exists a right continuous, non-decreasing $F $
and a subsequence $(F_{n_k})_k$ of $(F_n)_n$ where $F_n$ is and a subsequence $(F_{n_k})_k$ of $(F_n)_n$ where $F_n$ is
the probability distribution function of $\mu_n$, the probability distribution function of $\mu_n$,
@ -251,7 +251,7 @@ such that $F_{n_k}(x) \to F(x)$ for all $x$ where $F$ is continuous.
\mu_{n_k}\left( (- \infty, x] \right) = F_{n_k}(x) \to F(x). \mu_{n_k}\left( (- \infty, x] \right) = F_{n_k}(x) \to F(x).
\] \]
Again, given $\epsilon > 0$, there exists $A > 0$, such that Again, given $\epsilon > 0$, there exists $A > 0$, such that
$\mu_{n_k}\left( (-A,A) \right) > 1 - 2 \epsilon$ (\autoref{levyproofc1}). $\mu_{n_k}\left( (-A,A) \right) > 1 - 2 \epsilon$ (\yaref{levyproofc1}).
Hence $F(x) \ge 1 - 2 \epsilon$ for $x > A $ Hence $F(x) \ge 1 - 2 \epsilon$ for $x > A $
and $F(x) \le 2\epsilon$ for $x < -A$. and $F(x) \le 2\epsilon$ for $x < -A$.
@ -262,13 +262,13 @@ Since $F$ is a probability distribution function, there exists
a probability measure $\nu$ on $\R$ such that $F$ is the distribution a probability measure $\nu$ on $\R$ such that $F$ is the distribution
function of $\nu$. function of $\nu$.
Since $F_{n_k}(x) \to F_n(x)$ at all continuity points $x$ of $F$, Since $F_{n_k}(x) \to F_n(x)$ at all continuity points $x$ of $F$,
by \autoref{lec10_thm1} we obtain that by \yaref{lec10_thm1} we obtain that
$\mu_{n_k} \overset{k \to \infty}{\implies} \nu$. $\mu_{n_k} \overset{k \to \infty}{\implies} \nu$.
Hence Hence
$\phi_{\mu_{n_k}}(t) \to \phi_\nu(t)$, by the other direction of that theorem. $\phi_{\mu_{n_k}}(t) \to \phi_\nu(t)$, by the other direction of that theorem.
But by assumption, But by assumption,
$\phi_{\mu_{n_k}}(\cdot ) \to \phi_n(\cdot )$ so $\phi_{\mu}(\cdot) = \phi_{\nu}(\cdot )$. $\phi_{\mu_{n_k}}(\cdot ) \to \phi_n(\cdot )$ so $\phi_{\mu}(\cdot) = \phi_{\nu}(\cdot )$.
By \autoref{charfuncuniqueness}, we get $\mu = \nu$. By the \yaref{charfuncuniqueness}, we get $\mu = \nu$.
We have shown, that $\mu_{n_k} \implies \mu$ along a subsequence. We have shown, that $\mu_{n_k} \implies \mu$ along a subsequence.
We still need to show that $\mu_n \implies \mu$. We still need to show that $\mu_n \implies \mu$.
@ -281,7 +281,7 @@ We still need to show that $\mu_n \implies \mu$.
% \notes % \notes
% \end{subproof} % \end{subproof}
Assume that $\mu_n$ does not converge to $\mu$. Assume that $\mu_n$ does not converge to $\mu$.
By \autoref{lec10_thm1}, pick a continuity point $x_0$ of $F$, By \yaref{lec10_thm1}, pick a continuity point $x_0$ of $F$,
such that $F_n(x_0) \not\to F(x_0)$. such that $F_n(x_0) \not\to F(x_0)$.
Pick $\delta > 0$ and a subsequence $F_{n_1}(x_0), F_{n_2}(x_0), \ldots$ Pick $\delta > 0$ and a subsequence $F_{n_1}(x_0), F_{n_2}(x_0), \ldots$
which are all outside $(F(x_0) - \delta, F(x_0) + \delta)$. which are all outside $(F(x_0) - \delta, F(x_0) + \delta)$.

View File

@ -89,7 +89,7 @@ We now want to generalize this to arbitrary random variables.
\subsection{Existence of Conditional Probability} \subsection{Existence of Conditional Probability}
We will give two different proves of \autoref{conditionalexpectation}. We will give two different proves of \yaref{conditionalexpectation}.
The first one will use orthogonal projections. The first one will use orthogonal projections.
The second will use the Radon-Nikodym theorem. The second will use the Radon-Nikodym theorem.
We'll first do the easy proof, derive some properties We'll first do the easy proof, derive some properties
@ -139,7 +139,7 @@ and then do the harder proof.
$K$ is closed, since a pointwise limit of $\cG$-measurable $K$ is closed, since a pointwise limit of $\cG$-measurable
functions is $\cG$ measurable (if it exists). functions is $\cG$ measurable (if it exists).
By \autoref{orthproj}, By \yaref{orthproj},
there exists $z \in K$ such that there exists $z \in K$ such that
\[\bE[(X - Z)^2] = \inf \{ \bE[(X- W)^2] ~|~ W \in L^2(\cG)\}\] \[\bE[(X - Z)^2] = \inf \{ \bE[(X- W)^2] ~|~ W \in L^2(\cG)\}\]
and and
@ -168,5 +168,6 @@ and then do the harder proof.
Define $Z(\omega) \coloneqq \limsup_{n \to \infty} Z_n(\omega)$. Define $Z(\omega) \coloneqq \limsup_{n \to \infty} Z_n(\omega)$.
Then $Z$ is $\cG$-measurable and since $Z_n \uparrow Z$, Then $Z$ is $\cG$-measurable and since $Z_n \uparrow Z$,
by MCT, $\bE(Z \One_G) = \bE(X \One_G)$ for all $G \in \cG$. by the \yaref{cmct},
$\bE(Z \One_G) = \bE(X \One_G)$ for all $G \in \cG$.
\end{refproof} \end{refproof}

View File

@ -5,7 +5,7 @@ We want to derive some properties of conditional expectation.
\begin{theorem}[Law of total expectation] \begin{theorem}[Law of total expectation]
\label{ceprop1} \label{ceprop1}
\label{totalexpectation} \yalabel{Law of Total Expectation}{Total Expectation}{totalexpectation}
\[ \[
\bE[\bE[X | \cG ]] = \bE[X]. \bE[\bE[X | \cG ]] = \bE[X].
\] \]
@ -26,7 +26,7 @@ We want to derive some properties of conditional expectation.
\[ \[
\int_A X \dif \bP \ge \frac{1}{n}\bP(A) + \int_A Y \dif \bP, \int_A X \dif \bP \ge \frac{1}{n}\bP(A) + \int_A Y \dif \bP,
\] \]
contradicting property (b) from \autoref{conditionalexpectation}. contradicting property (b) from \yaref{conditionalexpectation}.
\end{proof} \end{proof}
\begin{example} \begin{example}
@ -37,7 +37,7 @@ We want to derive some properties of conditional expectation.
\begin{theorem}[Linearity] \begin{theorem}[Linearity]
\label{ceprop3} \label{ceprop3}
\label{celinearity} \yalabel{Linearity of Conditional Expectation}{Linearity}{celinearity}
For all $a,b \in \R$ For all $a,b \in \R$
we have we have
\[ \[
@ -50,7 +50,7 @@ We want to derive some properties of conditional expectation.
\begin{theorem}[Positivity] \begin{theorem}[Positivity]
\label{ceprop4} \label{ceprop4}
\label{cpositivity} \yalabel{Positivity of Conditional Expectation}{Positivity}{cpositivity}
If $X \ge 0$, then $\bE[X | \cG] \ge 0$ a.s. If $X \ge 0$, then $\bE[X | \cG] \ge 0$ a.s.
\end{theorem} \end{theorem}
\begin{proof} \begin{proof}
@ -65,7 +65,7 @@ We want to derive some properties of conditional expectation.
\end{proof} \end{proof}
\begin{theorem}[Conditional monotone convergence theorem] \begin{theorem}[Conditional monotone convergence theorem]
\label{ceprop5} \label{ceprop5}
\label{mcmt} \yalabel{Conditional Monotone Converence Theorem}{MCT}{cmct}
Let $X_n,X \in L^1(\Omega, \cF, \bP)$. Let $X_n,X \in L^1(\Omega, \cF, \bP)$.
Suppose $X_n \ge 0$ with $X_n \uparrow X$. Suppose $X_n \ge 0$ with $X_n \uparrow X$.
Then $\bE[X_n|\cG] \uparrow \bE[X|\cG]$. Then $\bE[X_n|\cG] \uparrow \bE[X|\cG]$.
@ -73,7 +73,7 @@ We want to derive some properties of conditional expectation.
\begin{proof} \begin{proof}
Let $Z_n$ be a version of $\bE[X_n | Y]$. Let $Z_n$ be a version of $\bE[X_n | Y]$.
Since $X_n \ge 0$ and $X_n \uparrow$, Since $X_n \ge 0$ and $X_n \uparrow$,
by \autoref{cpositivity}, by the \yaref{cpositivity},
we have we have
\[ \[
\bE[X_n | \cG] \overset{\text{a.s.}}{\ge } 0 \bE[X_n | \cG] \overset{\text{a.s.}}{\ge } 0
@ -100,7 +100,7 @@ We want to derive some properties of conditional expectation.
\begin{theorem}[Conditional Fatou] \begin{theorem}[Conditional Fatou]
\label{ceprop6} \label{ceprop6}
\label{cfatou} \yalabel{Conditional Fatou's Lemma}{Fatou}{cfatou}
Let $X_n \in L^1(\Omega, \cF, \bP)$, $X_n \ge 0$. Let $X_n \in L^1(\Omega, \cF, \bP)$, $X_n \ge 0$.
Then Then
\[ \[
@ -112,7 +112,7 @@ We want to derive some properties of conditional expectation.
\end{proof} \end{proof}
\begin{theorem}[Conditional dominated convergence theorem] \begin{theorem}[Conditional dominated convergence theorem]
\label{ceprop7} \label{ceprop7}
\label{cdct} \yalabel{Conditional Dominated Convergence Theorem}{DCT}{cdct}
Let $X_n,Y \in L^1(\Omega, \cF, \bP)$. Let $X_n,Y \in L^1(\Omega, \cF, \bP)$.
Suppose that $\sup_n |X_n(\omega)| < Y(\omega)$ a.e.~ Suppose that $\sup_n |X_n(\omega)| < Y(\omega)$ a.e.~
and that $X_n$ converges to a pointwise limit $X$. and that $X_n$ converges to a pointwise limit $X$.
@ -124,7 +124,7 @@ We want to derive some properties of conditional expectation.
Recall Recall
\begin{fact}[Jensen's inequality] \begin{fact}[Jensen's inequality]
\label{jensen} \yalabel{Jensen's Inequality}{Jensen}{jensen}
If $c : \R \to \R$ is convex and $\bE[|c \circ X|] < \infty$, If $c : \R \to \R$ is convex and $\bE[|c \circ X|] < \infty$,
then $\bE[c \circ X] \overset{\text{a.s.}}{\ge} c(\bE[X])$. then $\bE[c \circ X] \overset{\text{a.s.}}{\ge} c(\bE[X])$.
\end{fact} \end{fact}
@ -132,7 +132,7 @@ Recall
For conditional expectation, we have For conditional expectation, we have
\begin{theorem}[Conditional Jensen's inequality] \begin{theorem}[Conditional Jensen's inequality]
\label{ceprop8} \label{ceprop8}
\label{cjensen} \yalabel{Jensen's Inequality}{Jensen}{cjensen}
Let $X \in L^1(\Omega, \cF, \bP)$. Let $X \in L^1(\Omega, \cF, \bP)$.
If $c : \R \to \R$ is convex and $\bE[|c \circ X|] < \infty$, If $c : \R \to \R$ is convex and $\bE[|c \circ X|] < \infty$,
then $\bE[c \circ X | \cG] \ge c(\bE[X | \cG])$ a.s. then $\bE[c \circ X | \cG] \ge c(\bE[X | \cG])$ a.s.
@ -147,7 +147,7 @@ For conditional expectation, we have
\] \]
\end{fact} \end{fact}
\begin{refproof}{cjensen} \begin{refproof}{cjensen}
By \autoref{convapprox}, $c(x) \ge a_n X + b_n$ By \yaref{convapprox}, $c(x) \ge a_n X + b_n$
for all $n$. for all $n$.
Hence Hence
\[ \[
@ -159,12 +159,13 @@ For conditional expectation, we have
we conclude that a.s~this happens simultaneously for all $n$. we conclude that a.s~this happens simultaneously for all $n$.
Hence Hence
\[ \[
\bE[c(X) | \cG] \ge \sup_n (a_n \bE[X | \cG] + b_n) \overset{\text{\autoref{convapprox}}}{=} c(\bE(X | \cG)). \bE[c(X) | \cG] \ge \sup_n (a_n \bE[X | \cG] + b_n) \overset{\yaref{convapprox}}{=} c(\bE(X | \cG)).
\] \]
\end{refproof} \end{refproof}
Recall Recall
\begin{fact}[Hölder's inequality] \begin{fact}[Hölder's inequality]
\yalabel{Hölder's Inequality}{Hölder}{thm:hoelder}
Let $p,q \ge 1$ such that $\frac{1}{p} + \frac{1}{q} = 1$. Let $p,q \ge 1$ such that $\frac{1}{p} + \frac{1}{q} = 1$.
Suppose $X \in L^p(\bP)$ and $Y \in L^q(\bP)$. Suppose $X \in L^p(\bP)$ and $Y \in L^q(\bP)$.
Then Then
@ -175,7 +176,7 @@ Recall
\begin{theorem}[Conditional Hölder's inequality] \begin{theorem}[Conditional Hölder's inequality]
\label{ceprop9} \label{ceprop9}
\label{choelder} \yalabel{Hölder's Inequality}{Hölder}{choelder}
Let $p,q \ge 1$ such that $\frac{1}{p} + \frac{1}{q} = 1$. Let $p,q \ge 1$ such that $\frac{1}{p} + \frac{1}{q} = 1$.
Suppose $X \in L^p(\bP)$ and $Y \in L^q(\bP)$. Suppose $X \in L^p(\bP)$ and $Y \in L^q(\bP)$.
Then Then
@ -203,7 +204,7 @@ Recall
\begin{theorem}[Tower property] \begin{theorem}[Tower property]
\label{ceprop10} \label{ceprop10}
\label{cetower} \yalabel{Tower Property}{Tower}{cetower}
Suppose $\cF \supset \cG \supset \cH$ are sub-$\sigma$-algebras. Suppose $\cF \supset \cG \supset \cH$ are sub-$\sigma$-algebras.
Then Then
\[ \[
@ -245,7 +246,7 @@ Assume $Y = \One_B$, then $Y$ simple, then take the limit (using that $Y$ is bou
\begin{theorem}[Role of independence] \begin{theorem}[Role of independence]
\label{ceprop12} \label{ceprop12}
\label{ceroleofindependence} \yalabel{Role of Independence}{Independence}{ceroleofindependence}
Let $X$ be a random variable, Let $X$ be a random variable,
and let $\cG, \cH$ be $\sigma$-algebras. and let $\cG, \cH$ be $\sigma$-algebras.
@ -272,10 +273,10 @@ Assume $Y = \One_B$, then $Y$ simple, then take the limit (using that $Y$ is bou
For $\bE[S_{n+1} | \cF_n]$ we obtain For $\bE[S_{n+1} | \cF_n]$ we obtain
\begin{IEEEeqnarray*}{rCl} \begin{IEEEeqnarray*}{rCl}
\bE[S_{n+1} | \cF_n] &\overset{\text{\autoref{celinearity}}}{=}& \bE[S_{n+1} | \cF_n] &\overset{\yaref{celinearity}}{=}&
\bE[S_n | \cF_n] + \bE[X_{n+1} | \cF_n]\\ \bE[S_n | \cF_n] + \bE[X_{n+1} | \cF_n]\\
&\overset{\text{a.s.}}{=}& S_n + \bE[X_{n+1} | \cF_n]\\ &\overset{\text{a.s.}}{=}& S_n + \bE[X_{n+1} | \cF_n]\\
&\overset{\text{\autoref{ceprop12}}}{=}& S_{n} + \bE[X_n]\\ &\overset{\yaref{ceroleofindependence}}{=}& S_{n} + \bE[X_n]\\
&=& S_n. &=& S_n.
\end{IEEEeqnarray*} \end{IEEEeqnarray*}
\end{example} \end{example}

View File

@ -48,7 +48,7 @@ Note that in this setting, if $\mu(A) = 0$ it follows that $\nu(A) = 0$.
The Radon Nikodym theorem is the converse of that: The Radon Nikodym theorem is the converse of that:
\begin{theorem}[Radon-Nikodym] \begin{theorem}[Radon-Nikodym]
\label{radonnikodym} \yalabel{Radon-Nikodym Theorem}{Radon-Nikodym}{radonnikodym}
Let $\mu$ and $\nu$ be two $\sigma$-finite measures Let $\mu$ and $\nu$ be two $\sigma$-finite measures
on $(\Omega, \cF)$. on $(\Omega, \cF)$.
@ -85,14 +85,14 @@ The Radon Nikodym theorem is the converse of that:
\end{definition} \end{definition}
With \autoref{radonnikodym} we get a very short proof of the existence With the \yaref{radonnikodym} we get a very short proof of the existence
of conditional expectation: of conditional expectation:
\begin{proof}[Second proof of \autoref{conditionalexpectation}] \begin{proof}[Second proof of \yaref{conditionalexpectation}]
Let $(\Omega, \cF, \bP)$ as always, $X \in L^1(\bP)$ and $\cG \subseteq \cF$. Let $(\Omega, \cF, \bP)$ as always, $X \in L^1(\bP)$ and $\cG \subseteq \cF$.
It suffices to consider the case of $X \ge 0$. It suffices to consider the case of $X \ge 0$.
For all $G \in \cG$, define $\nu(G) \coloneqq \int_G X \dif \bP$. For all $G \in \cG$, define $\nu(G) \coloneqq \int_G X \dif \bP$.
Obviously, $\nu \ll \bP$ on $\cG$. Obviously, $\nu \ll \bP$ on $\cG$.
Then apply \autoref{radonnikodym}. Then apply the \yaref{radonnikodym}.
\end{proof} \end{proof}
@ -212,7 +212,7 @@ Typically $\cF_n = \sigma(X_1, \ldots, X_n)$ for a sequence of random variables.
Likewise, if $f$ is concave, then $((f(X_n))_n$ is a supermartingale. Likewise, if $f$ is concave, then $((f(X_n))_n$ is a supermartingale.
\end{corollary} \end{corollary}
\begin{proof} \begin{proof}
Apply \autoref{cjensen}. Apply \yaref{cjensen}.
\end{proof} \end{proof}
\begin{corollary} \begin{corollary}

View File

@ -130,9 +130,9 @@ exists pointwise.
\end{lemma} \end{lemma}
\begin{proof} \begin{proof}
Since $C_n \ge 0$, Since $C_n \ge 0$,
by \autoref{lem:gambling-strategy} we have that $Y_n$ is a supermartingale. by \yaref{lem:gambling-strategy} we have that $Y_n$ is a supermartingale.
Hence $\bE[Y_N] \le \bE[Y_1] = 0$. Hence $\bE[Y_N] \le \bE[Y_1] = 0$.
From \autoref{lec17l2} it follows that From \yaref{lec17l2} it follows that
\[ \[
(b-a) \bE[U_N([a,b])] \le \bE[Y_n] + \bE[(X_N-a)^-] \le \bE[(X_N-a)^-]. (b-a) \bE[U_N([a,b])] \le \bE[Y_n] + \bE[(X_N-a)^-] \le \bE[(X_N-a)^-].
\] \]
@ -146,7 +146,7 @@ exists pointwise.
In particular, $\bP[U_\infty = \infty] = 0$. In particular, $\bP[U_\infty = \infty] = 0$.
\end{corollary} \end{corollary}
\begin{proof} \begin{proof}
By \autoref{lec17l3} By \yaref{lec17l3}
we have that we have that
\[(b-a) \bE[U_N([a,b])] \le \bE[ | X_N| ] + |a| \le \sup_n \bE[|X_n|] + |a|.\] \[(b-a) \bE[U_N([a,b])] \le \bE[ | X_N| ] + |a| \le \sup_n \bE[|X_n|] + |a|.\]
Since $U_N(\cdot) \ge 0$ and $U_N(\cdot ) \uparrow U_\infty(\cdot )$, Since $U_N(\cdot) \ge 0$ and $U_N(\cdot ) \uparrow U_\infty(\cdot )$,
@ -160,8 +160,8 @@ Let us now consider the case that our process $(X_n)_{n \ge 1}$ is a supermartin
bounded in $L^1(\bP)$. bounded in $L^1(\bP)$.
\begin{theorem}[Doob's martingale convergence theorem] \begin{theorem}[Doob's martingale convergence theorem]
\label{doobmartingaleconvergence} \yalabel{Doob's Martingale Convergence Theorem}{Doob}{doobmartingaleconvergence}
\label{doob} \yalabel{Doob's Martingale Convergence Theorem}{Doob}{doob}
Any supermartingale bounded in $L^1$ converges almost surely to a Any supermartingale bounded in $L^1$ converges almost surely to a
random variable, which is almost surely finite. random variable, which is almost surely finite.
In particular, any non-negative supermartingale converges a.s.~to a finite random variable. In particular, any non-negative supermartingale converges a.s.~to a finite random variable.
@ -179,8 +179,8 @@ We have
\end{IEEEeqnarray*} \end{IEEEeqnarray*}
We have $\Lambda_{a,b} \subseteq \{\omega : U_{\infty}([a,b])(\omega) = \infty\}$ We have $\Lambda_{a,b} \subseteq \{\omega : U_{\infty}([a,b])(\omega) = \infty\}$
by \autoref{lec17l1}. by \yaref{lec17l1}.
By \autoref{lec17l3} we have $\bP(\Lambda_{a,b}) = 0$, By \yaref{lec17l3} we have $\bP(\Lambda_{a,b}) = 0$,
hence $\bP(\Lambda) = 0$. hence $\bP(\Lambda) = 0$.
Thus there exists a random variable $X_\infty$ such that $X_n \xrightarrow{a.s.} X_\infty$. Thus there exists a random variable $X_\infty$ such that $X_n \xrightarrow{a.s.} X_\infty$.
@ -192,7 +192,7 @@ Thus there exists a random variable $X_\infty$ such that $X_n \xrightarrow{a.s.}
We have. We have.
\begin{IEEEeqnarray*}{rCl} \begin{IEEEeqnarray*}{rCl}
\bE[|X_\infty|] &=& \bE[\liminf_{n \to \infty} |X_n|]\\ \bE[|X_\infty|] &=& \bE[\liminf_{n \to \infty} |X_n|]\\
&\overset{\text{Fatou}}{\le }& \liminf_n \bE[|X_n|]\\ &\overset{\yaref{cfatou}}{\le }& \liminf_n \bE[|X_n|]\\
&\le & \sup_n \bE[|X_n|]\\ &\le & \sup_n \bE[|X_n|]\\
&<& \infty. &<& \infty.
\end{IEEEeqnarray*} \end{IEEEeqnarray*}

View File

@ -37,7 +37,7 @@ Hence the same holds for submartingales, i.e.
&=& X_n. &=& X_n.
\end{IEEEeqnarray*} \end{IEEEeqnarray*}
By \autoref{doobmartingaleconvergence}, By \yaref{doobmartingaleconvergence},
there exists an a.s.~limit $X_\infty$. there exists an a.s.~limit $X_\infty$.
By the SLLN, we have almost surely By the SLLN, we have almost surely
\[ \[
@ -116,7 +116,7 @@ consider $L^2$.
\[ \[
\bE[X_n^2] = \bE[X_0^2] + \sum_{j=1}^{n} \bE[Y_j^2] \bE[X_n^2] = \bE[X_0^2] + \sum_{j=1}^{n} \bE[Y_j^2]
\] \]
by \autoref{martingaleincrementsorthogonal}. by \yaref{martingaleincrementsorthogonal}.
In particular, In particular,
\[ \[
\sup_n \bE[X_n^2] < \infty \iff \sum_{j=1}^{\infty} \bE[Y_j^2] < \infty. \sup_n \bE[X_n^2] < \infty \iff \sum_{j=1}^{\infty} \bE[Y_j^2] < \infty.
@ -124,7 +124,7 @@ consider $L^2$.
Since $(X_n)_n$ is bounded in $L^2$, Since $(X_n)_n$ is bounded in $L^2$,
there exists $X_\infty$ such that $X_n \xrightarrow{\text{a.s.}} X_\infty$ there exists $X_\infty$ such that $X_n \xrightarrow{\text{a.s.}} X_\infty$
by \autoref{doob}. by \yaref{doob}.
It remains to show $X_n \xrightarrow{L^2} X_\infty$. It remains to show $X_n \xrightarrow{L^2} X_\infty$.
For any $r \in \N$, consider For any $r \in \N$, consider
@ -143,7 +143,7 @@ consider $L^2$.
Now let $p \ge 1$ be not necessarily $2$. Now let $p \ge 1$ be not necessarily $2$.
First, we need a very important inequality: First, we need a very important inequality:
\begin{theorem}[Doob's $L^p$ inequality] \begin{theorem}[Doob's $L^p$ inequality]
\label{dooblp} \yalabel{Doob's Martingale Inequalities}{Doob}{dooblp}
Suppose that $(X_n)_n$ is a martingale Suppose that $(X_n)_n$ is a martingale
or a non-negative submartingale. or a non-negative submartingale.
Let $X_n^\ast \coloneqq \max \{|X_1|, |X_2|, \ldots, |X_n|\}$ Let $X_n^\ast \coloneqq \max \{|X_1|, |X_2|, \ldots, |X_n|\}$
@ -158,7 +158,7 @@ First, we need a very important inequality:
\end{enumerate} \end{enumerate}
\end{theorem} \end{theorem}
In order to prove \autoref{dooblp}, we first need In order to prove \yaref{dooblp}, we first need
\begin{lemma} \begin{lemma}
\label{dooplplemma} \label{dooplplemma}
Let $p > 1$ and $X,Y$ non-negative random variables Let $p > 1$ and $X,Y$ non-negative random variables
@ -182,7 +182,7 @@ In order to prove \autoref{dooblp}, we first need
&=& \int Y(\omega)^p \dif \bP(\omega)\\ &=& \int Y(\omega)^p \dif \bP(\omega)\\
&=&\int_{\Omega} \left( \int_0^{Y(\omega)} p \ell^{p-1} \dif \ell &=&\int_{\Omega} \left( \int_0^{Y(\omega)} p \ell^{p-1} \dif \ell
\right) \dif \bP(\omega)\\ \right) \dif \bP(\omega)\\
&\overset{\text{Fubini}}{=}& &\overset{\yaref{thm:fubini}}{=}&
\int_0^\infty p \ell^{p-1}\underbrace{\int_\Omega \One_{Y \ge \ell}\dif \bP}_% \int_0^\infty p \ell^{p-1}\underbrace{\int_\Omega \One_{Y \ge \ell}\dif \bP}_%
{\bP[Y \ge \ell]} \dif\ell. \label{l18star} {\bP[Y \ge \ell]} \dif\ell. \label{l18star}
\end{IEEEeqnarray} \end{IEEEeqnarray}
@ -192,7 +192,7 @@ In order to prove \autoref{dooblp}, we first need
\eqref{l18star} \eqref{l18star}
&\le& \int_0^\infty p \ell^{p-2} &\le& \int_0^\infty p \ell^{p-2}
\int_{\{Y(\omega) \ge \ell\}} X(\omega) \bP(\dif \omega)\dif \ell\\ \int_{\{Y(\omega) \ge \ell\}} X(\omega) \bP(\dif \omega)\dif \ell\\
&\overset{\text{Fubini}}{=}& &\overset{\yaref{thm:fubini}}{=}&
\int_\Omega X(\omega) \int_{0}^{Y(\omega)} p \ell^{p-2} \dif \ell\bP(\dif \omega)\\ \int_\Omega X(\omega) \int_{0}^{Y(\omega)} p \ell^{p-2} \dif \ell\bP(\dif \omega)\\
&=& \frac{p}{p-1} \int_{\omega} X(\omega) Y (\omega)^{p-1} \bP(\dif \omega)\\ &=& \frac{p}{p-1} \int_{\omega} X(\omega) Y (\omega)^{p-1} \bP(\dif \omega)\\
&\overset{\text{Hölder}}{\le}& \frac{p}{p-1} \|X\|_{L^p} \|Y\|_{p}^{p-1}, &\overset{\text{Hölder}}{\le}& \frac{p}{p-1} \|X\|_{L^p} \|Y\|_{p}^{p-1},
@ -212,11 +212,11 @@ In order to prove \autoref{dooblp}, we first need
\] \]
Then Then
\begin{equation} \begin{equation}
\bP[E_j] \overset{\text{Markov}}{\le } \frac{1}{\ell} \int_{E_j} |X_j| \dif \bP \bP[E_j] \overset{\yaref{thm:markov}}{\le } \frac{1}{\ell} \int_{E_j} |X_j| \dif \bP
\label{lec18eq2star} \label{lec18eq2star}
\end{equation} \end{equation}
We have that $(|X_n|)_n$ is a submartingale, We have that $(|X_n|)_n$ is a submartingale,
by \autoref{cor:convexmartingale} by \yaref{cor:convexmartingale}
in the case of $X_n$ being a martingale in the case of $X_n$ being a martingale
and trivially if $X_n$ is non-negative. and trivially if $X_n$ is non-negative.
Hence Hence
@ -225,7 +225,7 @@ In order to prove \autoref{dooblp}, we first need
&=& \One_{E_j} \bE[(|X_n| - |X_{j}|)|\cF_j]\\ &=& \One_{E_j} \bE[(|X_n| - |X_{j}|)|\cF_j]\\
&\overset{\text{a.s.}}{\ge }& 0. &\overset{\text{a.s.}}{\ge }& 0.
\end{IEEEeqnarray*} \end{IEEEeqnarray*}
By the law of total expectation, \autoref{totalexpectation}, By the \yaref{totalexpectation},
it follows that it follows that
\begin{equation} \begin{equation}
\bE[\One_{E_j} (|X_n| - |X_j|)] \ge 0. \label{lec18eq3star} \bE[\One_{E_j} (|X_n| - |X_j|)] \ge 0. \label{lec18eq3star}
@ -241,6 +241,6 @@ In order to prove \autoref{dooblp}, we first need
This proves the first part. This proves the first part.
For the second part, we apply the first part and For the second part, we apply the first part and
\autoref{dooplplemma} (choose $Y \coloneqq X_n^\ast$). \yaref{dooplplemma} (choose $Y \coloneqq X_n^\ast$).
\end{refproof} \end{refproof}
\todo{Branching process} \todo{Branching process}

View File

@ -57,10 +57,10 @@ However, some subsets can be easily described, e.g.
\sup_n \underbrace{\bP[|X_n| > K]^{\frac{1}{q}}}_% \sup_n \underbrace{\bP[|X_n| > K]^{\frac{1}{q}}}_%
{\le K^{-\frac{1}{q}} \bE[|X_n|]^{\frac{1}{q}}}\\ {\le K^{-\frac{1}{q}} \bE[|X_n|]^{\frac{1}{q}}}\\
\end{IEEEeqnarray*} \end{IEEEeqnarray*}
where we have applied Markov's inequality. % TODO REF where we have applied \yaref{thm:markov}.
Since $\sup_n \bE[|X_n|^{1+\delta}] < \infty$, Since $\sup_n \bE[|X_n|^{1+\delta}] < \infty$,
we have that $\sup_n \bE[|X_n|] < \infty$ by Jensen (\autoref{jensen}). we have that $\sup_n \bE[|X_n|] < \infty$ by \yaref{jensen}.
Hence for $K$ large enough relevant term is less than $\epsilon$. Hence for $K$ large enough relevant term is less than $\epsilon$.
\end{proof} \end{proof}
@ -93,7 +93,7 @@ However, some subsets can be easily described, e.g.
but $\int_{F_n} |X| \dif \bP \ge \epsilon$. but $\int_{F_n} |X| \dif \bP \ge \epsilon$.
Since $\sum_{n} \bP(F_n) < \infty$, Since $\sum_{n} \bP(F_n) < \infty$,
by \autoref{borelcantelli}, by \yaref{thm:borelcantelli},
\[\bP[\underbrace{\limsup_n F_n}_{\text{\reflectbox{$\coloneqq$}}F}] = 0.\] \[\bP[\underbrace{\limsup_n F_n}_{\text{\reflectbox{$\coloneqq$}}F}] = 0.\]
We have We have
\begin{IEEEeqnarray*}{rCl} \begin{IEEEeqnarray*}{rCl}
@ -108,7 +108,7 @@ However, some subsets can be easily described, e.g.
This yields a contradiction since $\bP(F) = 0$. This yields a contradiction since $\bP(F) = 0$.
\item We want to apply part (a) to $F = \{ |X| > k\}$. \item We want to apply part (a) to $F = \{ |X| > k\}$.
By Markov, $\bP(F) \le \frac{1}{k} \bE[|X|]$. By \yaref{thm:markov}, $\bP(F) \le \frac{1}{k} \bE[|X|]$.
Since $\bE[|X|] < \infty$, we can choose $k$ large enough Since $\bE[|X|] < \infty$, we can choose $k$ large enough
to get $\bP(F) \le \delta$. to get $\bP(F) \le \delta$.
\end{enumerate} \end{enumerate}
@ -120,7 +120,7 @@ However, some subsets can be easily described, e.g.
\[ \[
\bE[|X_n| \One_{|X_n| > k}] \le \bE[|Y| \One_{|Y| > k}] < \epsilon \bE[|X_n| \One_{|X_n| > k}] \le \bE[|Y| \One_{|Y| > k}] < \epsilon
\] \]
for $k$ large enough by \autoref{lec19f4} (b). for $k$ large enough by \yaref{lec19f4} (b).
\end{refproof} \end{refproof}
\begin{fact}\label{lec19f5} \begin{fact}\label{lec19f5}
@ -135,9 +135,9 @@ However, some subsets can be easily described, e.g.
\label{lec19eqstar} \label{lec19eqstar}
\end{equation} \end{equation}
Let $Y = \bE[X | \cG]$ for some sub-$\sigma$-algebra $\cG$. Let $Y = \bE[X | \cG]$ for some sub-$\sigma$-algebra $\cG$.
Then, by \autoref{cjensen}, $|Y| \le \bE[ |X| | \cG]$. Then, by \yaref{cjensen}, $|Y| \le \bE[ |X| | \cG]$.
Hence $\bE[|Y|] \le \bE[|X|]$. Hence $\bE[|Y|] \le \bE[|X|]$.
By Markov's inequality, By \yaref{thm:markov},
it follows that $\bP[|Y| > k] < \delta$ it follows that $\bP[|Y| > k] < \delta$
for $k > \frac{\bE[|X|]}{\delta}$. for $k > \frac{\bE[|X|]}{\delta}$.
Note that $\{|Y| > k\} \in \cG$. Note that $\{|Y| > k\} \in \cG$.
@ -179,19 +179,19 @@ However, some subsets can be easily described, e.g.
+ \int |\phi(X_n) - \phi(X)| \dif \bP\\ + \int |\phi(X_n) - \phi(X)| \dif \bP\\
\end{IEEEeqnarray*} \end{IEEEeqnarray*}
We have $\int_{|X_n| > k} \underbrace{|X_n - \phi(X_n)|}_{\le |X_n| + | \phi(X_n)| \le 2 |X_n|} \dif \bP\le \epsilon$ by uniform integrability and We have $\int_{|X_n| > k} \underbrace{|X_n - \phi(X_n)|}_{\le |X_n| + | \phi(X_n)| \le 2 |X_n|} \dif \bP\le \epsilon$ by uniform integrability and
\autoref{lec19f4} part (b). \yaref{lec19f4} part (b).
Similarly $\int_{|X| > k} |X - \phi(X)| \dif \bP < \epsilon$. Similarly $\int_{|X| > k} |X - \phi(X)| \dif \bP < \epsilon$.
Since $\phi$ is Lipschitz, Since $\phi$ is Lipschitz,
$ X_n \xrightarrow{\bP} X \implies \phi(X_n) \xrightarrow{\bP} \phi(X)$. $ X_n \xrightarrow{\bP} X \implies \phi(X_n) \xrightarrow{\bP} \phi(X)$.
By the bounded convergence theorem, \autoref{thm:boundedconvergence}, By the \yaref{thm:boundedconvergence}
$|\phi(X_n)| \le k \implies \int | \phi(X_n) - \phi(X)| \dif \bP \to 0$. $|\phi(X_n)| \le k \implies \int | \phi(X_n) - \phi(X)| \dif \bP \to 0$.
(1) $\implies$ (2) (1) $\implies$ (2)
$X_n \xrightarrow{L^1} X \implies X_n \xrightarrow{\bP} X$ $X_n \xrightarrow{L^1} X \implies X_n \xrightarrow{\bP} X$
by Markov's inequality (see \autoref{claim:convimpll1p}). by \yaref{thm:markov} (see \yaref{claim:convimpll1p}).
Fix $\epsilon > 0$. Fix $\epsilon > 0$.
We have We have
@ -202,8 +202,8 @@ However, some subsets can be easily described, e.g.
\end{IEEEeqnarray*} \end{IEEEeqnarray*}
for all $\delta > 0$ and suitable $k$. for all $\delta > 0$ and suitable $k$.
Hence $\bP[|X_n| > k] < \delta$ by Markov's inequality. Hence $\bP[|X_n| > k] < \delta$ by \yaref{thm:markov}.
Then by \autoref{lec19f4} part (a) it follows that Then by \yaref{lec19f4} part (a) it follows that
\[ \[
\int_{|X_n| > k} |X_n| \dif \bP \le \underbrace{\int |X - X_n| \dif \bP}_{< \epsilon} + \int_{|X_n| > k} |X| \dif \bP \le 2 \epsilon. \int_{|X_n| > k} |X_n| \dif \bP \le \underbrace{\int |X - X_n| \dif \bP}_{< \epsilon} + \int_{|X_n| > k} |X| \dif \bP \le 2 \epsilon.
\] \]

View File

@ -1,16 +1,16 @@
\lecture{20}{2023-06-27}{} \lecture{20}{2023-06-27}{}
\begin{refproof}{ceismartingale} \begin{refproof}{ceismartingale}
By the tower property (\autoref{cetower}) By the \yaref{cetower}
it is clear that $(\bE[X | \cF_n])_n$ it is clear that $(\bE[X | \cF_n])_n$
is a martingale. is a martingale.
First step: First step:
Assume that $X$ is bounded. Assume that $X$ is bounded.
Then, by \autoref{cjensen}, $|X_n| \le \bE[|X| | \cF_n]$, Then, by \yaref{cjensen}, $|X_n| \le \bE[|X| | \cF_n]$,
hence $\sup_{\substack{n \in \N \\ \omega \in \Omega}} | X_n(\omega)| < \infty$. hence $\sup_{\substack{n \in \N \\ \omega \in \Omega}} | X_n(\omega)| < \infty$.
Thus $(X_n)_n$ is a martingale in $L^{\infty} \subseteq L^2$. Thus $(X_n)_n$ is a martingale in $L^{\infty} \subseteq L^2$.
By the convergence theorem for martingales in $L^2$ By the convergence theorem for martingales in $L^2$
(\autoref{martingaleconvergencel2}) (\yaref{martingaleconvergencel2})
there exists a random variable $Y$, there exists a random variable $Y$,
such that $X_n \xrightarrow{L^2} Y$. such that $X_n \xrightarrow{L^2} Y$.
@ -74,10 +74,10 @@
Thus $X_n \xrightarrow{L^p} X$. Thus $X_n \xrightarrow{L^p} X$.
\end{refproof} \end{refproof}
For the proof of \autoref{martingaleisce}, For the proof of \yaref{martingaleisce},
we need the following theorem, which we won't prove here: we need the following theorem, which we won't prove here:
\begin{theorem}[Banach Alaoglu] \begin{theorem}[Banach Alaoglu]
\label{banachalaoglu} \yalabel{Banach Alaoglu}{Banach Alaoglu}{banachalaoglu}
Let $X$ be a normed vector space and $X^\ast$ its Let $X$ be a normed vector space and $X^\ast$ its
continuous dual. continuous dual.
Then the closed unit ball in $X^\ast$ is compact Then the closed unit ball in $X^\ast$ is compact
@ -96,7 +96,7 @@ we need the following theorem, which we won't prove here:
\end{fact} \end{fact}
\begin{refproof}{martingaleisce} \begin{refproof}{martingaleisce}
Since $(X_n)_n$ is bounded in $L^p$, by \autoref{banachalaoglu}, Since $(X_n)_n$ is bounded in $L^p$, by \yaref{banachalaoglu},
there exists $X \in L^p$ and a subsequence there exists $X \in L^p$ and a subsequence
$(X_{n_k})_k$ such that for all $Y \in L^q$ ($\frac{1}{p} + \frac{1}{q} = 1$ ) $(X_{n_k})_k$ such that for all $Y \in L^q$ ($\frac{1}{p} + \frac{1}{q} = 1$ )
\[ \[
@ -115,7 +115,7 @@ we need the following theorem, which we won't prove here:
&\overset{\text{for }n_k \ge m}{=}& \bE[X_m \One_A]. &\overset{\text{for }n_k \ge m}{=}& \bE[X_m \One_A].
\end{IEEEeqnarray*} \end{IEEEeqnarray*}
Hence $X_n = \bE[X | \cF_m]$ by the uniqueness of conditional expectation Hence $X_n = \bE[X | \cF_m]$ by the uniqueness of conditional expectation
and by \autoref{ceismartingale}, and by \yaref{ceismartingale},
we get the convergence. we get the convergence.
\end{refproof} \end{refproof}
@ -298,7 +298,7 @@ we need the following theorem, which we won't prove here:
\end{example} \end{example}
\begin{theorem}[Optional Stopping] \begin{theorem}[Optional Stopping]
\label{optionalstopping} \yalabel{Optional Stopping Theorem}{Optional Stopping}{optionalstopping}
Let $(X_n)_n$ be a supermartingale Let $(X_n)_n$ be a supermartingale
and let $T$ be a stopping time and let $T$ be a stopping time
taking values in $\N$. taking values in $\N$.
@ -320,7 +320,7 @@ we need the following theorem, which we won't prove here:
$\bE[X_T] = \bE[X_0]$. $\bE[X_T] = \bE[X_0]$.
\end{theorem} \end{theorem}
\begin{proof} \begin{proof}
(i) was already done in \autoref{roptionalstoppingi}. (i) was already done in \yaref{roptionalstoppingi}.
(ii): Since $(X_n)_n$ is bounded, we get that (ii): Since $(X_n)_n$ is bounded, we get that
\begin{IEEEeqnarray*}{rCl} \begin{IEEEeqnarray*}{rCl}
@ -345,3 +345,17 @@ we need the following theorem, which we won't prove here:
applying this to $(X_n)_n$ and $(-X_n)_n$, applying this to $(X_n)_n$ and $(-X_n)_n$,
which are both supermartingales. which are both supermartingales.
\end{proof} \end{proof}
\begin{remark}+
Let $(X_n)_n$ be a supermartingale and $T$ a stopping time.
If $(X_n)_n$ itself is not bounded,
but $T$ ensures boundedness,
i.e. $T < \infty$ a.s.~and $(X_{T \wedge n})_n$
is uniformly bounded,
the \yaref{optionalstopping} can still be applied, as
\[
\bE[X_T] = \bE[X_{T \wedge T}]
\overset{\yaref{optionalstopping}}{\le} \bE[X_{T \wedge 0}]
= \bE[X_0].
\]
\end{remark}

View File

@ -6,7 +6,7 @@ This is the last lecture relevant for the exam.
\begin{goal} \begin{goal}
We want to see an application of the We want to see an application of the
optional stopping theorem \ref{optionalstopping}. \ref{optionalstopping}.
\end{goal} \end{goal}
\begin{notation} \begin{notation}
@ -110,7 +110,7 @@ is the unique solution to this problem.
% We have $\sigma(\One_{X_{n+1} \in B}) \subseteq \sigma(X_{n}, \xi_{n+1})$. % We have $\sigma(\One_{X_{n+1} \in B}) \subseteq \sigma(X_{n}, \xi_{n+1})$.
% $\sigma(X_1,\ldots,X_{n-1})$ % $\sigma(X_1,\ldots,X_{n-1})$
% is independent of $\sigma( \sigma(\One_{X_{n+1} \in B}), X_n)$. % is independent of $\sigma( \sigma(\One_{X_{n+1} \in B}), X_n)$.
% Hence the claim follows from \autoref{ceroleofindependence}. % Hence the claim follows from \yaref{ceroleofindependence}.
\end{subproof} \end{subproof}
\end{example} \end{example}

View File

@ -4,7 +4,7 @@
\subsubsection{Construction of iid random variables.} \subsubsection{Construction of iid random variables.}
\begin{itemize} \begin{itemize}
\item Definition of a consistent family (\autoref{def:consistentfamily}) \item Definition of a consistent family (\yaref{def:consistentfamily})
\item Important construction: \item Important construction:
Consider a distribution function $F$ and define Consider a distribution function $F$ and define
@ -16,22 +16,22 @@
\item Examples of consistent and inconsistent families \item Examples of consistent and inconsistent families
\todo{Exercises} \todo{Exercises}
\item Kolmogorov's consistency theorem \item Kolmogorov's consistency theorem
(\autoref{thm:kolmogorovconsistency}) (\yaref{thm:kolmogorovconsistency})
\end{itemize} \end{itemize}
\subsubsection{Limit theorems} \subsubsection{Limit theorems}
\begin{itemize} \begin{itemize}
\item Work with iid.~random variables. \item Work with iid.~random variables.
\item Notions of convergence (\autoref{def:convergence}) \item Notions of convergence (\yaref{def:convergence})
\item Implications between different notions of convergence (very important) and counter examples. \item Implications between different notions of convergence (very important) and counter examples.
(\autoref{thm:convergenceimplications}) (\yaref{thm:convergenceimplications})
\item Laws of large numbers: (\autoref{lln}) \item Laws of large numbers: (\yaref{lln})
\begin{itemize} \begin{itemize}
\item WLLN: convergence in probability \item WLLN: convergence in probability
\item SLLN: weak convergence \item SLLN: weak convergence
\end{itemize} \end{itemize}
\item \autoref{thm2} (building block for SLLN): \item \yaref{thm2} (building block for SLLN):
Let $(X_n)$ be independent with mean $0$ and $\sum \sigma_n^2 < \infty$, Let $(X_n)$ be independent with mean $0$ and $\sum \sigma_n^2 < \infty$,
then $ \sum X_n $ converges a.s. then $ \sum X_n $ converges a.s.
\begin{itemize} \begin{itemize}
@ -43,12 +43,12 @@
$\sum \frac{\pm 1}{ n^{\frac{1}{2} -\epsilon}}$ does not converge a.s.~for any $\epsilon > 0$. $\sum \frac{\pm 1}{ n^{\frac{1}{2} -\epsilon}}$ does not converge a.s.~for any $\epsilon > 0$.
\end{itemize} \end{itemize}
\item Kolmogorov's inequality (\autoref{thm:kolmogorovineq}) \item \yaref{thm:kolmogorovineq}
\item Kolmogorov's $0-1$-law. (\autoref{kolmogorov01}) \item \yaref{kolmogorov01}
In particular, a series of independent random variables converges with probability $0$ or $1$. In particular, a series of independent random variables converges with probability $0$ or $1$.
\item Kolmogorov's 3 series theorem. (\autoref{thm:kolmogorovthreeseries}) \item \yaref{thm:kolmogorovthreeseries}
\begin{itemize} \begin{itemize}
\item What are those $3$ series? \item What are those $3$ series?
\item Applications \item Applications
@ -59,15 +59,15 @@
\begin{itemize} \begin{itemize}
\item Definition of Fourier transform \item Definition of Fourier transform
(\autoref{def:characteristicfunction}) (\yaref{def:characteristicfunction})
\item The Fourier transform uniquely determines the probability distribution. \item The Fourier transform uniquely determines the probability distribution.
It is bounded, so many theorems are easily applicable. It is bounded, so many theorems are easily applicable.
\item Uniqueness theorem (\autoref{charfuncuniqueness}), \item \yaref{charfuncuniqueness},
inversion formula (\autoref{inversionformula}), ... \yaref{inversionformula}, ...
\item Levy's continuity theorem (\autoref{levycontinuity}), \item \yaref{levycontinuity},
(\autoref{genlevycontinuity}) \yaref{genlevycontinuity}
\item Bochner's theorem for positive definite function (\autoref{thm:bochner}) \item \yaref{thm:bochner}
\item Bochner's theorem for the mass at a point (\autoref{bochnersformula}) \item \yaref{bochnersformula}
\item Related notions \item Related notions
\todo{TODO} \todo{TODO}
\begin{itemize} \begin{itemize}
@ -81,7 +81,7 @@
\paragraph{Weak convergence} \paragraph{Weak convergence}
\begin{itemize} \begin{itemize}
\item Definition of weak convergence % ( test against continuous, bounded functions). \item Definition of weak convergence % ( test against continuous, bounded functions).
(\autoref{def:weakconvergence}) (\yaref{def:weakconvergence})
\item Examples: \item Examples:
\begin{itemize} \begin{itemize}
\item $(\delta_{\frac{1}{n}})_n$, \item $(\delta_{\frac{1}{n}})_n$,
@ -93,9 +93,8 @@
\item Non-examples: $(\delta_n)_n$ \item Non-examples: $(\delta_n)_n$
\item How does one prove weak convergence? How does one write this down in a clear way? \item How does one prove weak convergence? How does one write this down in a clear way?
\begin{itemize} \begin{itemize}
\item \autoref{lec10_thm1}, \item \yaref{lec10_thm1},
\item Levy's continuity theorem \item \yaref{levycontinuity},
\ref{levycontinuity},
\item Generalization of Levy's continuity theorem \item Generalization of Levy's continuity theorem
\ref{genlevycontinuity} \ref{genlevycontinuity}
\end{itemize} \end{itemize}
@ -111,12 +110,12 @@
\subsubsubsection{CLT} \subsubsubsection{CLT}
\begin{itemize} \begin{itemize}
\item Statement of the CLT \item Statement of the \yaref{clt}
\item Several versions: \item Several versions:
\begin{itemize} \begin{itemize}
\item iid (\autoref{clt}), \item iid,
\item Lindeberg (\autoref{lindebergclt}), \item \yaref{lindebergclt},
\item Lyapanov (\autoref{lyapunovclt}) \item \yaref{lyapunovclt}
\end{itemize} \end{itemize}
\item How to apply this? Exercises! \item How to apply this? Exercises!
\end{itemize} \end{itemize}
@ -124,11 +123,11 @@
\subsubsection{Conditional expectation} \subsubsection{Conditional expectation}
\begin{itemize} \begin{itemize}
\item Definition and existence of conditional expectation for $X \in L^1(\Omega, \cF, \bP)$ \item Definition and existence of conditional expectation for $X \in L^1(\Omega, \cF, \bP)$
(\autoref{conditionalexpectation}) (\yaref{conditionalexpectation})
\item If $H = L^2(\Omega, \cF, \bP)$, then $\bE[ \cdot | \cG]$ \item If $H = L^2(\Omega, \cF, \bP)$, then $\bE[ \cdot | \cG]$
is the (unique) projection on the closed subspace $L^2(\Omega, \cG, \bP)$. is the (unique) projection on the closed subspace $L^2(\Omega, \cG, \bP)$.
Why is this a closed subspace? Why is the projection orthogonal? Why is this a closed subspace? Why is the projection orthogonal?
\item Radon-Nikodym Theorem \ref{radonnikodym} \item \yaref{radonnikodym}
(Proof not relevant for the exam) (Proof not relevant for the exam)
\item (Non-)examples of mutually absolutely continuous measures \item (Non-)examples of mutually absolutely continuous measures
Singularity in this context? % TODO Singularity in this context? % TODO
@ -137,30 +136,30 @@
\subsubsection{Martingales} \subsubsection{Martingales}
\begin{itemize} \begin{itemize}
\item Definition of Martingales (\autoref{def:martingale}) \item Definition of Martingales (\yaref{def:martingale})
\item Doob's convergence theorem (\autoref{doobmartingaleconvergence}), \item Doob's convergence theorem (\yaref{doobmartingaleconvergence}),
Upcrossing inequality (\autoref{lec17l1}, \autoref{lec17l2}, \autoref{lec17l3}) Upcrossing inequality (\yaref{lec17l1}, \yaref{lec17l2}, \yaref{lec17l3})
(downcrossings for submartingales) (downcrossings for submartingales)
\item Examples of Martingales converging a.s.~but not in $L^1$ \item Examples of Martingales converging a.s.~but not in $L^1$
(\autoref{ex:martingale-not-converging-in-l1}) (\yaref{ex:martingale-not-converging-in-l1})
\item Bounded in $L^2$ $\implies$ convergence in $L^2$ \item Bounded in $L^2$ $\implies$ convergence in $L^2$
(\autoref{martingaleconvergencel2}). (\yaref{martingaleconvergencel2}).
\item Martingale increments are orthogonal in $L^2$! \item Martingale increments are orthogonal in $L^2$!
(\autoref{martingaleincrementsorthogonal}) (\yaref{martingaleincrementsorthogonal})
\item Doob's (sub-)martingale inequalities \item Doob's (sub-)martingale inequalities
(\autoref{dooblp}), (\yaref{dooblp}),
\item $\bP[\sup_{k \le n} M_k \ge x]$ $\leadsto$ Look at martingale inequalities! \item $\bP[\sup_{k \le n} M_k \ge x]$ $\leadsto$ Look at martingale inequalities!
Estimates might come from Doob's inequalities if $(M_k)_k$ is a (sub-)martingale. Estimates might come from Doob's inequalities if $(M_k)_k$ is a (sub-)martingale.
\item Doob's $L^p$ convergence theorem \item Doob's $L^p$ convergence theorem
(\autoref{ceismartingale}, \autoref{martingaleisce}). (\yaref{ceismartingale}, \yaref{martingaleisce}).
\begin{itemize} \begin{itemize}
\item Why is $p > 1$ important? \textbf{Role of Banach-Alaoglu} \item Why is $p > 1$ important? \textbf{Role of \yaref{banachalaoglu}}
\item This is an important proof. \item This is an important proof.
\end{itemize} \end{itemize}
\item Uniform integrability (\autoref{def:ui}) \item Uniform integrability (\yaref{def:ui})
\item What are stopping times? (\autoref{def:stopping-time}) \item What are stopping times? (\yaref{def:stopping-time})
\item (Non-)examples of stopping times \item (Non-)examples of stopping times
\item \textbf{Optional stopping theorem} (\autoref{optionalstopping}) \item \textbf{\yaref{optionalstopping}}
- be really comfortable with this. - be really comfortable with this.
\end{itemize} \end{itemize}

View File

@ -35,7 +35,7 @@ from the lecture on stochastic.
This notion of convergence was actually This notion of convergence was actually
defined during the course of the lecture, defined during the course of the lecture,
but has been added here for completeness; but has been added here for completeness;
see \autoref{def:weakconvergence}. see \yaref{def:weakconvergence}.
} }
($X_n \xrightarrow{\text{d}} X$) ($X_n \xrightarrow{\text{d}} X$)
iff for every continuous, bounded $f: \R \to \R$ iff for every continuous, bounded $f: \R \to \R$
@ -128,7 +128,7 @@ from the lecture on stochastic.
\begin{subproof} \begin{subproof}
Let $F$ be the distribution function of $X$ Let $F$ be the distribution function of $X$
and $(F_n)_n$ the distribution functions of $(X_n)_n$. and $(F_n)_n$ the distribution functions of $(X_n)_n$.
By \autoref{lec10_thm1} By \yaref{lec10_thm1}
it suffices to show that $F_n(t) \to F(t)$ for all continuity it suffices to show that $F_n(t) \to F(t)$ for all continuity
points $t$ of $F$. points $t$ of $F$.
Let $t$ be a continuity point of $F$. Let $t$ be a continuity point of $F$.
@ -155,7 +155,7 @@ from the lecture on stochastic.
\label{claim:convimplpl1} \label{claim:convimplpl1}
$X_n \xrightarrow{\bP} X \notimplies X_n\xrightarrow{L^1} X$.% $X_n \xrightarrow{\bP} X \notimplies X_n\xrightarrow{L^1} X$.%
\footnote{Note that the implication holds under certain assumptions, \footnote{Note that the implication holds under certain assumptions,
see \autoref{thm:l1iffuip}.} see \yaref{thm:l1iffuip}.}
\end{claim} \end{claim}
\begin{subproof} \begin{subproof}
Take $([0,1], \cB([0,1 ]), \lambda)$ Take $([0,1], \cB([0,1 ]), \lambda)$
@ -170,7 +170,7 @@ from the lecture on stochastic.
$X_n \xrightarrow{\text{a.s.}} X \notimplies X_n\xrightarrow{L^1} X$. $X_n \xrightarrow{\text{a.s.}} X \notimplies X_n\xrightarrow{L^1} X$.
\end{claim} \end{claim}
\begin{subproof} \begin{subproof}
We can use the same counterexample as in \autoref{claim:convimplpl1} We can use the same counterexample as in \yaref{claim:convimplpl1}
$\bP[\lim_{n \to \infty} X_n = 0] \ge \bP[X_n = 0] = 1 - \frac{1}{n} \to 0$. $\bP[\lim_{n \to \infty} X_n = 0] \ge \bP[X_n = 0] = 1 - \frac{1}{n} \to 0$.
We have already seen, that $X_n$ does not converge in $L_1$. We have already seen, that $X_n$ does not converge in $L_1$.
@ -220,7 +220,7 @@ from the lecture on stochastic.
\end{refproof} \end{refproof}
\begin{theorem}[Bounded convergence theorem] \begin{theorem}[Bounded convergence theorem]
\label{thm:boundedconvergence} \yalabel{Bounded Convergence Theorem}{Bounded convergence}{thm:boundedconvergence}
Suppose that $X_n \xrightarrow{\bP} X$ and there exists Suppose that $X_n \xrightarrow{\bP} X$ and there exists
some $K$ such that $|X_n| \le K$ for all $n$. some $K$ such that $|X_n| \le K$ for all $n$.
Then $X_n \xrightarrow{L^1} X$. Then $X_n \xrightarrow{L^1} X$.
@ -262,7 +262,7 @@ from the lecture on stochastic.
\begin{theorem}+[Riemann-Lebesgue] \begin{theorem}+[Riemann-Lebesgue]
%\footnote{see exercise 3.3} %\footnote{see exercise 3.3}
\label{riemann-lebesgue} \yalabel{Riemann-Lebesgue}{Riemann-Lebesgue}{riemann-lebesgue}
Let $f: \R \to \R$ be integrable. Let $f: \R \to \R$ be integrable.
Then Then
\[ \[
@ -271,6 +271,7 @@ from the lecture on stochastic.
\end{theorem} \end{theorem}
\begin{theorem}+[Fubini-Tonelli] \begin{theorem}+[Fubini-Tonelli]
\yalabel{Fubuni-Tonelli}{Fubini}{thm:fubini}
%\footnote{exercise sheet 1} %\footnote{exercise sheet 1}
Let $(\Omega_{i}, \cF_i, \bP_i), i \in \{0,1\}$ Let $(\Omega_{i}, \cF_i, \bP_i), i \in \{0,1\}$
be probability spaces and $\Omega \coloneqq \Omega_0 \otimes \Omega_1$, be probability spaces and $\Omega \coloneqq \Omega_0 \otimes \Omega_1$,
@ -296,6 +297,7 @@ from the lecture on stochastic.
This is taken from section 6.1 of the notes on Stochastik. This is taken from section 6.1 of the notes on Stochastik.
\begin{theorem}[Markov's inequality] \begin{theorem}[Markov's inequality]
\yalabel{Markov's Inequality}{Markov}{thm:markov}
Let $X$ be a random variable and $a > 0$. Let $X$ be a random variable and $a > 0$.
Then Then
\[ \[
@ -311,6 +313,7 @@ This is taken from section 6.1 of the notes on Stochastik.
\end{proof} \end{proof}
\begin{theorem}[Chebyshev's inequality] \begin{theorem}[Chebyshev's inequality]
\yalabel{Chebyshev's Inequality}{Chebyshev}{thm:chebyshev}
Let $X$ be a random variable and $a > 0$. Let $X$ be a random variable and $a > 0$.
Then Then
\[ \[
@ -322,14 +325,14 @@ This is taken from section 6.1 of the notes on Stochastik.
\begin{IEEEeqnarray*}{rCl} \begin{IEEEeqnarray*}{rCl}
\bP[|X-\bE(X)| \ge a] \bP[|X-\bE(X)| \ge a]
&=& \bP[|X - \bE(X)|^2 \ge a^2]\\ &=& \bP[|X - \bE(X)|^2 \ge a^2]\\
&\overset{\text{Markov}}{\le}& \frac{\bE[|X - \bE(X)|^2]}{a^2}. &\overset{\yaref{thm:markov}}{\le}& \frac{\bE[|X - \bE(X)|^2]}{a^2}.
\end{IEEEeqnarray*} \end{IEEEeqnarray*}
\end{proof} \end{proof}
How do we prove that something happens almost surely? How do we prove that something happens almost surely?
The first thing that should come to mind is: The first thing that should come to mind is:
\begin{lemma}[Borel-Cantelli] \begin{lemma}[Borel-Cantelli]
\label{borelcantelli} \yalabel{Borel-Cantelli}{Borel-Cantelli}{thm:borelcantelli}
If we have a sequence of events $(A_n)_{n \ge 1}$ If we have a sequence of events $(A_n)_{n \ge 1}$
such that $\sum_{n \ge 1} \bP(A_n) < \infty$, such that $\sum_{n \ge 1} \bP(A_n) < \infty$,
then $\bP[ A_n \text{for infinitely many $n$}] = 0$ then $\bP[ A_n \text{for infinitely many $n$}] = 0$

49
jrpie-yaref.sty Normal file
View File

@ -0,0 +1,49 @@
\NeedsTeXFormat{LaTeX2e}
\ProvidesPackage{jrpie-yaref}[2023/07/28 - yet another ref]
\RequirePackage{hyperref}
\RequirePackage{amstext}
\newcommand{\yaref@text@large}[1]{%
\ifcsname yaref@longlabel@#1\endcsname%
\hyperref[#1]{\csname yaref@longlabel@#1\endcsname\ \ref*{#1}}%
\else%
\autoref{#1}%
\fi%
}
\newcommand{\yaref@text@small}[1]{%
\ifcsname yaref@shortlabel@#1\endcsname%
\hyperref[#1]{\csname yaref@shortlabel@#1\endcsname}%
\else%
(\ref{#1})%
\fi%
}
\newcommand{\yaref@math@large}[1]{%
\text{\yaref@text@large{#1}}%
}
\newcommand{\yaref@math@small}[1]{%
\text{\yaref@text@small{#1}}%
}
\newcommand{\yaref@math@verysmall}[1]{%
\yaref@math@small{#1}%
}
\newcommand{\yalabel}[3]{%
\write\@auxout{\noexpand\expandafter\noexpand\gdef\noexpand\csname yaref@longlabel@#3\noexpand\endcsname{#1}}%
\write\@auxout{\noexpand\expandafter\noexpand\gdef\noexpand\csname yaref@shortlabel@#3\noexpand\endcsname{#2}}%
\expandafter\gdef\csname yaref@longlabel@#3\endcsname{#1}%
\expandafter\gdef\csname yaref@shortlabel@#3\endcsname{#2}%
\label{#3}%
}
\newcommand{\yaref}[1]{%
\relax\ifmmode%
\mathchoice
{\yaref@math@large{#1}} % display style
{\yaref@math@large{#1}} % text style
{\yaref@math@small{#1}} % script style
{\yaref@math@verysmall{#1}} % scriptscript style
\else%
\yaref@text@large{#1}%
\fi%
}

View File

@ -8,6 +8,7 @@
\usepackage[index]{mkessler-vocab} \usepackage[index]{mkessler-vocab}
\usepackage{mkessler-code} \usepackage{mkessler-code}
\usepackage{jrpie-math} \usepackage{jrpie-math}
\usepackage{jrpie-yaref}
\usepackage[normalem]{ulem} \usepackage[normalem]{ulem}
\usepackage{pdflscape} \usepackage{pdflscape}
\usepackage{longtable} \usepackage{longtable}